You are on page 1of 104

Answer & Explanation

SECTION 1
1.

Answer: Option D

In the following non planar graph number of


independent loop equations are

Explanation:
Distortion less
a=

RG and Z0 =

RG

A
8
B.12
.
C.7
D.5
Answer & Explanation

View Answer Workspace Report Discuss in


Forum
3.
A certain 8 bit uniform quantization PCM
system can accommodate a signal ranging
from - 1 V to + V. The rms value of the signal

Answer: Option D

is V . The signal to quantization noise ratio


is:

Explanation:
Number of independent loop equations are
given by,

A
30 dB
.
B.46.91 dB
C.40 dB
D
49.92 dB
.
Answer & Explanation

L=B-N+1
where L : No. of loop equations
B : No. of branches = 12

Answer: Option D
N : No. of nodes = 8
Explanation:
L = 12 - 8 + 1 = 5.
(SNR)dB = 1.76 + 6.02n
View Answer Workspace Report Discuss in
Forum

n=8

2.

Thus (SNR)dB = 1.76 + 6.02 x 8 = 49.92 dB.

A transmission line has a characteristic


impedance of 50 and a resistance of 0.1
/m, if the line is distortion less, the
attenuation constant (in Np/m) is

View Answer Workspace Report Discuss in


Forum
4.

A
500
.
C.0.014

B.5
D.0.002

A rectangular waveguide, in dominant TE


mode, has dimensions 10 cm x 15 cm. The cut

off frequency is
A.10 GHz
B.1 GHz
C.15 GHz
D.25 GHz
Answer & Explanation
Answer: Option B
A.i(t) = 0.5 - 0.125e-1000t A
B.i(t) = 1.5 - 0.125e-1000t A
C.i(t) = 0.5 - 0.5e-1000t A
D.i(t) = 0.375e-1000t A
Answer & Explanation

Explanation:
For TE10 mode

Answer: Option A
Explanation:
i(f) = 0.5, i(i) = 0.75
i(t) = Vr + (ii - ij)e-1/p = 0.5 - 0.125e-1000t .
View Answer Workspace Report Discuss in
Forum
5.
A radar receiver has a noise figure of 10 dB at
300 K having a bandwidth of 2.5 MHz. The
minimum power it can receive is
A
3.45 x 10-15 W
.
B.1.38 x 10-15 W
C.7.5 x 10-15 W
D
93.15 x 10-15 W
.
Answer & Explanation

7.
In a semiconductor material. The hole
concentration is found to be 2 x 2.5 x 1015 cm3
. If mobility of carriers is 0.13 m2/ v-s. Then
find the current density if electric field
intensity is 3.62 x 10-19
A
7.6237 x 10-4 A/cm2
.
B.7.6237 x 10-5 A/cm2
C.7.6237 x 10-3 A/cm2
D
none of these
.
Answer & Explanation

Answer: Option D
Explanation:
Pm = kT0f(F - 1) = 93.15 x 10

View Answer Workspace Report Discuss in


Forum

Answer: Option A
-15

W.

Explanation:
Current density J = E

6.
In the circuit shown, in switch S is open for a
long time and is closed at t = 0. The current
i(t) for t 0+ is

Where = conductivity
Given -> = 0.13 m2/v-s = 0.13 x 104 cm2/V

Only p = 0 and q = 0 result in


controllability
The system is uncontrollable for all values
C.
of p and q
D We cannot conclude about controllability
. from the given data
Answer & Explanation
B.

sec
P = 2.25 x 1015/cm3
We have, ni = 1.5 x 1010
Also n.p. =
n=

Answer: Option C
Explanation:

/p

Use the condition of controllability.


View Answer Workspace Report Discuss in
Forum
9.

= (1.6 x 10-19 x 0.13 x 104 x 2.25 x 1015) x

= (0.468) (4.5 x 1015)


= 2.106 x 1015 /cm
J = E
Current density = 2.106 x 1015 x 3.620 x 10-

A MOSFET has a threshold voltage of 1 V


and oxide thickness of 500 x 10-8 [r = 3.9; 0
= 8.85 x 10-14 F/cm, q = 1.6 x 10-19 c]. The
region under the gate is ion implanted for
threshold voltage tailoring. The base and type
of impant required to shift threshold voltage
to - 1 V are __________ .
A
8.6 x 1011/cm2, p-type
.
B.8.6 x 1011/cm2, n-type
C.0.86 x 109/cm2, p-type
D
1.02 x 1012/cm2, n-type
.
Answer & Explanation

19

Answer: Option A
-4

= 7.6237 x 10 A/m .

Explanation:

View Answer Workspace Report Discuss in


Forum
VT(new) = VT(odd) +

8.
Consider the system

with

and
where p and q are
arbitrary real numbers. Which of the
following statements about the controllability
of the system is true?
A The system is completely state controllable
. for any non zero values p and q

= 6.903 x 10-8

A
9
.
B.11.11
C.data not sufficient
D
10
.
Answer & Explanation

Answer: Option B

fB = - 8.6 x 1011

Explanation:

The threshold voltage is always negative for


p-channel and hence implant is of p-type.

Efficiency of antenna

View Answer Workspace Report Discuss in


Forum
Power gain = 10

10.
What is the propagation constant for air
filled wave guide with dimensions a = 1.59"
and b = 0.795" at 4.95 GHz?
A
0.6698
.

Thus directivity

View Answer Workspace Report Discuss in


Forum

B.0.7698

D
0.6598
.
Answer & Explanation

12.

C.0.503

If the power spectral density of stationary


random process is a sine-squared function of
frequency, the shape of its autocorrelation is

Answer: Option A
Explanation:

A
.

Here,
a = 1.59'' = 40.386 mm = 4.04 cm

B.

.
C.

11.
Radiation resistance of an antenna is 54
and loss resistance is 6 . If antenna has
power gain of 10, then directivity is:

14.
Consider the amplitude modulated (AM)
signal Ac cos ct + 2 cos mt cos ct For
demodulating the signal using envelope
detector, the minimum value of Ac should be

D
.

Answer & Explanation

A
2
.

Answer: Option B

B.1

D
0
.
Answer & Explanation
C.0.5

Explanation:
Since autocorrelation function and power
spectral density bears a Fourier transform
relation, then since required in frequency
domain will five rectangular convolutions in
time domain thus it is triangular function.

Answer: Option A
Explanation:
AC cos ct + 2 cos mt cos ct

View Answer Workspace Report Discuss in


Forum

AC cosct

13.
The inverse of given Laplace transform is

A
sin t
.
B.cos t
C.et
D 2t
e
.
Answer & Explanation
Answer: Option B
Explanation:

for envelope detection < 1


should be at least-2.

< 1 Ac

View Answer Workspace Report Discuss in


Forum
15.
In a 4 bit weighted resistor D/A converter,
the resistor value corresponding to LSB is
32 k. The resistor value corresponding to
MSB will be
A
32 K
.
B.16 K
C.8 K
D
4 K
.
Answer & Explanation

s = x2 - ex
Answer: Option D
Explanation:
x(t) = cos t.

2n - 1 R = 32 K&Omeg;

View Answer Workspace Report Discuss in


Forum

n=4

8R = 32

modulation?

R = 4 K.

A
x(t) = Acm(t)cos(2pfct)
.
B.x(t) = Ac [1 + m(t)]cos (2pfct)
C.
D x(t) = Accos(2pfmt)cos(2pfct) +
. Acsin(2pfmt)sin(2pfct)
Answer & Explanation

16.
Find 'X' in the circuit below :

Answer: Option C
Explanation:
f1(A, B, C, D) = (6, 7, 13, 14);
f2(A, B, C, D) = (3, 6, 7);
f3(A, B, C, D) = (5, 6, 7, 14, 15)
A p(0, 1, 2, 3, 4, 5, 6, 7, 8, 9, 10, 11, 12, 13,
. 15)
B.0
C.(14)
D
1
.
Answer & Explanation
Answer: Option C
Explanation:
f1(A, B, C, D) = (6, 7, 13, 14)
f2(A, B, C, D) = (3, 6, 7)
f1 f2 = (3, 13, 14)
f3 x (f1 f2) = (14) = y
X = f1 x y 14.

(C) is without over modulation.


View Answer Workspace Report Discuss in
Forum
18.
What are the values of Emax and Emin
displayed on the oscilloscope, when a 1 kV
P-P carries is modulated to 50%?
A
2 kV, 0.5 kV
.
B.1 kV, 0.5 kV
C.0.75 kV, 0.25 kV
D
0.5 kV, 1.5 kV
.
Answer & Explanation
Answer: Option C
Explanation:

View Answer Workspace Report Discuss in


Forum
17.
Suppose that the modulating signal is m(t) =
2cos (2pfmt) and the carrier signal is xC(t) =
AC cos(2pfct), which one of the following is
a conventional AM signal without over

Answer: Option B
Explanation:
Zs = Rs + (RB || Brs)
rc = 2.475 = 1.258 kV

Emax - Emin = 0.5 x 1 kV = 0.5 kV


2Emax = 1.5 kVi, Emax = 0.75 kV
Emin = 0.25 kV.
View Answer Workspace Report Discuss in
Forum

View Answer Workspace Report Discuss in


Forum
20.

19.

As the temperature increases the mobility of


electrons __________ .

Consider the common emitter amplifier


shown below with the following circuit
parameters: = 100, gm = 0.3861 A/V, r0 =
rp = 259 , Rs = 1 k, RB = 93 k, RC = 250
, RL = 1 kW, C1 = and C2 = 4.7mF.

A decreases because the number of carries


. increases with increased collisions
increases because the number of carries
B.
decreases with decreased collisions
C.remains same
D
none of the above
.
Answer & Explanation
Answer: Option A
Explanation:

Mobility of electrons
m = 2.5 for electrons and 2.7 for holes in Si.
The resistance seen by the source Vs is
A
258
.
B.1258
C.93 k
D

.
Answer & Explanation

21.
Consider the following
1. Maximum energy of electrons
liberated photoelectrically is
independent of light intensity

D
Decreased, Decreased, Decreased
.
Answer & Explanation

2. Maximum energy of electrons


liberated photoelectrically varies
nonlinearly with frequency of
incident light.

Answer: Option D

A.(1) is true and (2) is false


B.(1) and (2) both are true
C.(1) is false and (2) is true
D.(1) and (2) both are false
Answer & Explanation

Explanation:
Phase lag is delay between a correcting
signal of control system and response to it.
View Answer Workspace Report Discuss in
Forum

Answer: Option A
Explanation:

23.
The equivalent form of the logical
expression ABC + A BC + ABC + ABC +
AB C is

Statement (ii) is false because maximum


energy of photons varies linearly with
frequency of incident light. It can be shown
as follows :

A
C + (A B)
.
B.(A + B + C)(A + B + C)
C.(A + B + C)(A + B + C)(A + B + C)
D
(A + B + C)(A + B + C)(A + B + C)
.
Answer & Explanation
Answer: Option D

F1 > F2 > F3

Explanation:

j = light intensity = constant

This can be solve by using Boolean identity


but using k map it will be more easy

Photocurrent Vs Anode voltage with


frequency and incident light as a parameter.
The light intensity is constant.
View Answer Workspace Report Discuss in
Forum

(A + B + C) (A + B + C) (A + B + C).

22.
When phase-lag compensation is used in a
system, then gain cross over frequency,
bandwidth and undamped frequency are
respectively
A
Increased, Increased, Increased
.
B.Increased, Increased, Decreased
C.Increased, Decreased, Decreased

View Answer Workspace Report Discuss in


Forum
24.
The gradient of any scalar field always
yields __________ .
A
a solenoidal field
.

B.a conservative field


C.an irrotational field
D
none of these
.
Answer & Explanation
Answer: Option B

A
.

Explanation:

B.

Dot product is conservative. Gradient is


nothing but dot product.

C.

View Answer Workspace Report Discuss in


Forum

D
.
Answer & Explanation

In a flash type ADC, employing 15


comparators resolution with 10 volts
reference; is expected to be :

Answer: Option C

A
0.625 V
.
B.0.666 V
C.0.525 V
D
insufficient data
.
Answer & Explanation

P1 = abcd

25.

Explanation:

L1 = be, L2 = cf, L3 = dg
1 = 1

Answer: Option A
Explanation:

.
n

Since number of comparators = 2 - 1 = 15;

View Answer Workspace Report Discuss in


Forum

Then number of bits are '4'


27.

During transmission over a communicate


channel, bit errors occur independently with

Resolution

probability . If a block of 3 bits are


transmitted, the probability of at most one
bit error is equal to

26.
Consider the signal flow graph shown in
figure below. The gain

is :

A
.
B.
C.
D
none
.

Answer & Explanation


Then ROC for x1(n) + x2(n)
Answer: Option D
R1 R2(R1 < R2)
Explanation:
Probability of no-error + probability one
error

.
View Answer Workspace Report Discuss in
Forum

View Answer Workspace Report Discuss in


Forum
29.

28.
If x1[n]

Consider a unity feedback control system


with open-loop transfer function

x1(z) with ROC = R1 and

x2[n]
x2(z) with ROC = R2 then ROC
for x1[n] + x2[n] will be __________ (where
Ri Region of convergence and x1[n] and
x2[n] are causal).

The steady state error of the


system due to a unity step input is :
A
zero
.

A.R1 R2
B.R1 R2
C.(R1 R2) (R1)
D.(R1 R2) (R1)
Answer & Explanation

B.K

D
infinite
.
Answer & Explanation
C.1/K

Answer: Option A

Answer: Option A

Explanation:

Explanation:
.

R1 R2, Suppose R1 for x1(n) be

View Answer Workspace Report Discuss in


Forum
30.
The analog signal given below is sampled at
1200 samples per second. m(t) = 2 sin 120
pt + 3 sin 240 pt + 4 sin 360 pt + 5 sin 480
pt + 6 sin 600 pt + 7 sin 720 pt. The folding
(maximum) frequency is __________ .
Then ROC for x1(n) + x2(n)
R1 R2 (R1 < R2)

A
300 Hz
.
B.360 Hz
C.600 Hz

D
480 Hz
.
Answer & Explanation

x(t) + 4x(t)

Answer: Option C
Explanation:

+2

+5

+6=

+ 4x

This is required differential equation.


Folding frequency (max.) =
600 Hz.

View Answer Workspace Report Discuss in


Forum
32.
In the circuit below, delay of the EXOR and
AND

31.
If the system T.F. is
The differential
equation representing the system is
A
.
B.
C.
D
none of these
.
Answer & Explanation

Logic gates are 20 s and 10 s respectively,


then the wave y' and y (with initial condition
y = 1) will be :

Answer: Option C
Explanation:
A
.

(s3 + 2s2 + 5s + 6)y(s) = (s2 + 4) x (s)


s3y(s) + 2s2y(s) + 5sy(s) + 6y(s) = s2 x (s) + 4
x (s)
B.
Replacing s by
by x(t) we get

y(t) + 2

and y(s) by y(t) and x(s)

y(t) + 5

y(t) + 6y(t)

Explanation:
Ic = 8 A

C.

m = 0.8

D
.

Answer & Explanation

= 8 x 1.149 = 9.19 A.

Answer: Option B

View Answer Workspace Report Discuss in


Forum

Explanation:
Since y is at logic '1' initially, y' must be at
logic '1'.
Also, one input of the EXOR gate is tied to
zero therefore 'f' will be directly transferred
to the output.
The logic gates will introduce gate delays
which will lead to the answer .
View Answer Workspace Report Discuss in
Forum

34.
A noiseless 3 KHz channel can transmit
binary at the rate __________ .
A
12000 bps
.
B.10000 bps
C.6000 bps
D
3000 bps
.
Answer & Explanation
Answer: Option C
Explanation:

33.
The antenna current of an AM transmitter is
8 A, when only the carrier is sent, but it
increases to 8.93 A when the carrier is
modulated by a single sine wave. Determine
the antenna current when the percent of
modulation changes to 0.8
A
9.19 A
.
B.9.91 A
C.10.56 A
D
8.61 A
.
Answer & Explanation
Answer: Option A

Maximum data rate for noiseless channel is


given by Nyquist Theorem
= 2H log2 V bits/sec
V - discrete levels (Here Binary i.e. 2)
H - Bandwidth
Maximum data rate = 2H log2 V
= 2(3 KHz) log2 2 = 2 x 3 x 103 x 1
= 6 x 103 = 6000 bps.
View Answer Workspace Report Discuss in

Forum
Vz = 3 V.

35.
Find Vx Vy Vz
36.

If x[n]

then the sequence x[n] is

A
non-periodic
.
B.periodic
C.depends on n
D
none of these
.
Answer & Explanation

A
V = -6 Vy = 3 Vz = -3
. x
B.Vx = -6 Vy = -3 Vz = 1
C.Vx = 6 Vy = 3 Vz = 3
D
V = 6 Vy = 1 Vz = 3
. x
Answer & Explanation

Answer: Option A
Explanation:

Answer: Option D
Explanation:

Consider

Apply Mesh analysis

Number of samples per period = 10p


This is not a rational number. Hence signal
is non periodic.
View Answer Workspace Report Discuss in
Forum
37.

Apply KVL to mesh 2


-2 + Vy + 1 = 0

The impulse response and the excitation


function of a linear time invariant causal
system are shown in figure (a) and (b)
respectively. The output of the system at t =
2 sec is equal to

Vy = 1
Apply KVL to mesh 1
-8 + Vx + 2 = 0
Vx = 6 V
Apply KVL to mesh 3
-1 + Vz -2 = 0

A.0
B.2
C.
D.
Answer & Explanation

Answer: Option D

. say
Answer & Explanation

Explanation:

Answer: Option C
Explanation:
Statement 1 is false, since f{h(t)e3t}
corresponds to the value of the Laplace
transform of h(t) at s = 3.
If this converges, it implies that s = - 3 is in
the ROC.
A casual and stable system must always
have its ROC to the right of all its poles.
However, s = - 3 is not to the right of the
pole at s = - 2.

The area of the shaded region

Statement 2 is false, because it is equivalent


to stating that H(0) = 0. This contradicts the
fact that H(s) does not have a zero at the
origin.

.
View Answer Workspace Report Discuss in
Forum

Statement 3 is false. If h(t) is of finite


duration, then if its Laplace transform has
any points in its ROC, ROC must be the
entire s-plane.

38.
Consider a stable and causal system with
impulse response h(t) and system function
H(S). Suppose H(S) is rational, contains a
pole at S = - 2, and does not have a zero at
the origin. The location of all other poles
and zero is unknown for each of the
following statements. Let us determine
whether statement is true or false.

However, this is not consistent with H(s)


having a pole at s = - 2.
Statement 4 is false. If it were true, then
H(s) has a pole at s = - 2, it must also have a
pole at s = 2.
This is inconsistent with the fact that all the
poles of a causal and stable system must be
in the left half of the s-plane.

1. f[h(t) e-3t] converges

2.

View Answer Workspace Report Discuss in


Forum

3. h(t) has finite duration


4. H(s) = H(- s)
Choose correct option.
A
1 - True, 2 - False, 3 - True, 4 - False
.
B.1 - False, 2 - False, 3 - False, 4 - True
C.1 - False, 2 - False, 3 - False, 4 - False
D 1 - True, 2 - can't say, 3 - True, 4 - can't

39.
The specific gravity of tungsten is 13.8 and
its atomic weight is 184.0 Assume that there
are two free elements per atom. Then the
Fermi level or characteristic energy for the
crystal in eV will be
A
8.95 eV
.

B.-8.95 eV
C.7.326 eV
D
-7.326 eV
.
Answer & Explanation
Answer: Option C

A
1.5 x 108 m/s
.
B.5.5 x 108 m/s
C.3.46 x 108 m/s
D
0.18 x 107 m/s
.
Answer & Explanation

Explanation:

Answer: Option C

A quantity of any substance equal to its


molecular weight in grams is a mole of that
substance contains the same number of
molecules as one mole of any other
substance. This Avogadro's number and
equals 6.02 x 1023 molecules per mole. Thus

Explanation:

VP = 3.46 x 108 m/s.

41.
For a = - 1, find the value of V2

= 9.03 x 1028 electrons/m3


Since for tungsten the atomic and the
molecular weights are the same. Therefore
for tungsten
EF = 3.64 x 10-19()2/3
= 3.64 x 10- 19(9.03 x 1028)2/3
EF = 7.326 eV.
View Answer Workspace Report Discuss in
Forum
40.
An air-filled rectangular waveguide has
dimensions 8cm x 10cm . The phase velocity
of guided wave at frequency of 3 GHz for
TE10 mode is

.
B.0
C.V1
D
none of the above
.
Answer & Explanation
Answer: Option B
Explanation:
V1 = i1R1 + i2R2

i1 + ai1 = i2
.
V2 = i2R2
View Answer Workspace Report Discuss in
Forum
43.
For a given voltage, four heating coils will
produce maximum heat when connected
A
all in parallel
.
B.all in series
C.with two parallel pairs in series
D one pair in parallel with the other two in
. series
Answer & Explanation
Answer: Option A

Putting a = - 1

Explanation:
= 0 V.
View Answer Workspace Report Discuss in
Forum
42.
A unity-feedback control system has the
open loop transfer function
. If the input system is a unit ramp, the
steady state error will be
A
0
.

B.0.5

D
infinity
.
Answer & Explanation
C.2

Answer: Option C

View Answer Workspace Report Discuss in


Forum

Explanation:
44.

Denominator polynomial of a transfer


function of certain network is:
s3 + s2 + 2s + 24
Then the network is:

A
stable
.
B.oscillatory
C.unstable
D
depends on numerator polynomial
.
Answer & Explanation
Answer: Option C
C = 6 x 106 x 12 = 72 Mbps.

Explanation:
Routh array
46.

The ratio of the diffusion coefficient in a


semiconductor has the units
A -1
V
.

B.em.V-1

D
V.s
.
Answer & Explanation
C.V.cm-1
There is negative number present in first
column. Thus network is unstable.

Answer: Option A
Explanation:

View Answer Workspace Report Discuss in


Forum
45.

A TV picture is to be transmitted over a


channel of 6 MHz bandwidth at a 35 dB S/N
ratio. The capacity of the channel is
A
50 Mbps
.
B.100 Mbps
C.36 Mbps
D
72 Mbps
.
Answer & Explanation
Answer: Option D
Explanation:

View Answer Workspace Report Discuss in


Forum
47.
Consider the probability density f(x) = ae-b|x|
where x is a random variable whose
allowable values range from x = - to x = +
. P(1 x 2)
A
.
B.
C.
D -b -2b
(e - e )
.
Answer & Explanation

W = B = Bandwidth

Answer: Option C

Explanation:

P(x) =

ae-b|x|dx

to get

apply

aebxdx +

ae-bxdx,

we get = .
View Answer Workspace Report Discuss in
Forum
48.
The amplitude of a pair of composite
sinusoidal signal y(n) = x1(n) + x2(n) with
x1(n) = sin (5pn) x2(n) = 3 sin (5pn) is
__________ .
A
2
.

The input resistance Ri increases and the


magnitude of voltage gain Av decreases
The input resistance Ri decreases and the
B.
magnitude of voltage gain Av decreases
Both input resistance Ri and the
C.
magnitude of voltage gain Av decrease
Both input resistance Ri and the
D.
magnitude of voltage gain Av increase
Answer & Explanation
A.

B.3

D
1
.
Answer & Explanation
C.4

Answer: Option A

Answer: Option A

Explanation:
Amplitude =

Explanation:
Given circuit after removing CE will behave
as current-series feedback.

View Answer Workspace Report Discuss in


Forum

Overall voltage gain will decrease as


feedback signal comes into picture and since
it is current-series feedback, input
impedance increases.

49.
The amplifier circuit shown below uses a
silicon transistor. The capacitors Cc and CE
can be assumed to be short at signal
frequency and the effect of output resistance
r0 can be ignored. If CE is disconnected from
the circuit, which one of the following
statements is TRUE?

View Answer Workspace Report Discuss in


Forum
50.
A n-channel JFET has IDSS = 2 mA, Gate to
source voltage VGS = - 2 V and transconductance is 0.5 mW then pinch-off
voltage is __________ .
A-2V

.
B.2 V
C.- 4 V
D
4V
.
Answer & Explanation
Answer: Option C

SECTION 2
1.
If we manage to __________ our natural
resources, we would leave a better planet for
our children.

Explanation:

A
uphold
B.restrain
.
C.Cherish
D.conserve
Answer & Explanation
Answer: Option D
Explanation:
The clue in this sentence is 'If we manage to
__________ our natural resources' and 'better
planet'.

= - 8VP - 16
This implies that the blank should be filled by
a word which means 'preserve' or 'keep for
long time'.

+ 8VP + 16 = 0

(VP + 4)2 = 0

Therefore the word 'conserve' is the right


answer.

VP = -4 V.

View Answer Workspace Report Discuss in


Forum
2.
(2)3 + (3)4 = (?)5
A
4
.
B.11
C.none of these
D
not possible
.
Answer & Explanation
Answer: Option C
Explanation:
(2)3 + (3)4 = 2 + 3 = 5 and (10)5 = 51 + 0 = 5

.
Answer & Explanation

(2)3 + (3)4 = (10)5 .

Answer: Option A

View Answer Workspace Report Discuss in


Forum

Explanation:

3.
If g(t) = e- p t2 then G(0) is __________ (where
g(t) G(f)
A
.
B.unity
C.
D
impulse at f = 0
.
Answer & Explanation

Apply R-H.
View Answer Workspace Report Discuss in
Forum
5.
A discrete time linear shift-invariant system
has an impulse response h[n] with h[0] = 1,
h[1] = - 1, h[2] - 2, and zero otherwise. The
system is given an input sequence x[n] with
x[0] - x[2] - 1, and zero otherwise. The
number of nonzero samples in the output
sequence y[n], and the value of y[2] are,
respectively

Answer: Option B
Explanation:

A
5, 2
.
B.6, 2
C.6, 1
D
5, 3
.
Answer & Explanation

The signal g(t) given above is a Gaussian


pulse, and it satisfies the relation.

Therefore, its fourier transform is same as the


signal itself in frequency domain.

Answer: Option D

i.e. G(f) = e-pf2

Explanation:

Hence G(f)|f = 0 = e-p(0) = 1.


View Answer Workspace Report Discuss in
Forum
4.
The open loop transfer function of a unity
feedback system is:
The range of K for
which the system is stable is
A
.
B.13 > K > 0
C.
D -6 < K <

Use convolution to get the result.

6.
For a series resonant circuit at low frequency
circuit impedance is __________ and at high
frequency circuit impedance is __________
Fill in the blanks respectively
A
capacitive, inductive
.
B.inductive, capacitive
C.resistive, inductive
D
capacitive, resistive
.
Answer & Explanation

Answer: Option A
Explanation:

Characteristic equation:
So system is second order system of the two
poles are

From this curve; we can select option (a) as


correct answer.

View Answer Workspace Report Discuss in


Forum
8.
A signal with 10 V ranged and 1 KHz band
width is being digitized using a sample and
hold circuit and a 10 bit quantizer. The
minimum sampling rate is :

View Answer Workspace Report Discuss in


Forum
7.
The network is

A
.

A
62831.9 V/sec
.
B.125663.8 V/sec
C.31415.95 V/sec
D
None
.
Answer & Explanation
Answer: Option A

First order system and the pole

Explanation:
Maximum sampling rate = 2pfmax V.

Second order system of the two poles are


B.

View Answer Workspace Report Discuss in


Forum

Second order system of the poles is


C.
D
none of the above
.
Answer & Explanation
Answer: Option B
Explanation:
System T.F. =

9.
The circuit below represents function X(A, B,
C, D) as:

A.
A
(5, 8, 13, 14)
.
B.(0, 1, 2, 3, 4, 6, 7, 8, 10, 11, 12)
C.(5, 9, 13, 14)
D
(0, 1, 2, 3, 4, 5, 6, 7, 8, 10, 11, 12, 15)
.
Answer & Explanation

B.

Answer: Option C
Explanation:
X = A B C(0. D + 0.D) + A B C(0.D + 0.D) +
A B C(0.D + 1.D) + A B C(0.D + 0.D)

C.

+ A B C(0.D + 0.D) + A B C(0.D + 1.D) +


ABC(1.D + 0.D)
= (5, 9, 13, 14).
View Answer Workspace Report Discuss in
Forum

D.

10.
The truth table corresponding to the given
input digital gate :

Answer & Explanation


Answer: Option D
Explanation:
The circuit behaves like NAND gate.

system is s2 + 2s + 2, then the system is :


A
over damped
.

11 The z-transform of a signal


.

converges if

B
critically damped
.
C
under damped
.

and only if
A
.

D
undamped
.

B
.

Answer & Explanation

Answer: Option C

C
.

Explanation:

D
.

1 + G(s) H(s) = 0

Answer & Explanation

s2 + 2s + 2 = 0

Answer: Option A
Explanation:
< 1, under damped.
View Answer Workspace Report
Discuss in Forum

For u(n), a right handled sequence,

13 Given digits 2, 2, 3, 3, 3, 4, 4, 4, 4 how many


. distinct 4 digit numbers greater than 3000

can be formed?

|z| > , |z| =

A
50
.

B
51
.

|3z| > 1; |2z| > 1

C
52
.

D
54
.

Answer & Explanation

< 1;

< 1.

View Answer Workspace Report


Discuss in Forum

12 If the characteristic equation of a closed loop

Answer: Option B
Explanation:
The given digits are 2, 2, 3, 3, 3, 4, 4, 4, 4
we have to find the numbers that are greater

than 300

Discuss in Forum

The first digit can be 3 or 4 but not 2.


Now, let us fix the first, second and third
digits as 3, 2, 2, then the fourth place can be
filled in 3 ways.

The number of ways is 3 similarly, we fix


first third and fourth place as 3, 2 and 2
respectively (4) so the second place can be
filled in 3 ways again,

14 Consider circuit with 4 : 16 Demux below:


. Now:
1. f1 =

(2, 6, 11, 12)

2. f2 =

(2, 5, 14)

3.

= p(2, 3, 4, 5, 9, 13, 14)

The number of ways is 3


Now, we fix first, second and fourth,
previous cases and we obtain the same
result.
The number of ways is 3 so, the total
number of ways is 9 similarly this can done
by fixing the numbers as 3 and 4 (instead of
2) and thereby we obtain the a ways each
The number of numbers starting with 3 is 27
Similarly by taking 4 as the first digit we get
27 numbers
The number of numbers that are greater
than 3000 is 27 + 27 = 54

A
only 1 and 3 are correct
.
B
only 1 is correct
.
C
only 2 is correct
.
D
2 and 3 are correct
.
Answer & Explanation

But, 3222, 4222, is not possible as there are


only two 2's, 3333 is not possible as there are
only three 3's

Answer: Option D

The total number of numbers that are


greater than 3000 is 54 - 3 = 51.

f1 = y2 + y6 + y11 + y12

View Answer Workspace Report

Explanation:

and f2 = y1 + yl0 + yl3

f1(m, n, o, p) =

(3, 4, 9, 13)

B
f(A, B, C) =
.

f2(m, n, o, p) =

(2, 5, 14)

C
(A B)C
.

= (2, 3, 4, 5, 9, 13, 14).

(0, 1, 2, 3, 4, 5, 6, 7)

D
C
.

View Answer Workspace Report


Discuss in Forum

Answer & Explanation

Answer: Option D
15
.

Explanation:
The value of the integral

is

A
.
B
.
C
.

View Answer Workspace Report


Discuss in Forum

D
.
Answer & Explanation

Answer: Option A
Explanation:

17 A two port network is described by the


. relation

V1 = 2I1 + 3V2
I2 = - I1 + 2V2
Then Z-parameter of such network is

Compare this with


A
.

where s = 3.

16 The following circuit can be represented as :


.

B
.
C
.
D
.
Answer & Explanation

A
C
.

Answer: Option D
Explanation:

View Answer Workspace Report


Discuss in Forum

View Answer Workspace Report


Discuss in Forum

18 Find Y in the circuit below :


. f1(E, F) = (0, 1, 2, 3)

f2(A, B, E, F) = (0, 1, 2, 3, 5, 6, 9)
f3(A, B, E, F) = (4, 7, 8, 10, 11, 12, 13, 14,
15)

19 Calculate the stability factor and change in Ic


. from 25C to 100C for, = 50, RB/RE =

250, ICO = 19.9 nA for emitter bias


configuration
A
42.553, 0.85 F
.
B
40.91, 0.58 F
.
C
38.53, 0.85 F
.
D
41.10, 0.39 F
.
Answer & Explanation

A
f2
.

Answer: Option A

B
f2
.

Explanation:

C
0
.
D
(0, 1, 2, 3, 5, 6, 9)
.
Answer & Explanation

Answer: Option C
Explanation:

View Answer Workspace Report


Discuss in Forum

20 The star equivalent C1 C2, C3 of the delta

network is respectively

frequency R2 = 8f1 would be __________


[Neglect effect of 1 on the Beam width.]
A
R2 = R 1
.
B
R2 = 4R1
.
C
. R2 = 2

A
.

R1

D
. R2 =

B
.

Answer & Explanation

C
.

Answer: Option C
Explanation:

D
.

Answer: Option A

The range of the radar is directly


proportional to the square root of the
frequency.

Explanation:

Answer & Explanation

R1
and R2

21 In a radar system, the range of R1 is achieved

at a frequency f1. Then the range R2 at

View Answer Workspace Report


Discuss in Forum

22 With a line charge along z-axis the field

Answer: Option D
intensity is E =
; which of the
following figure shows an exact field
distribution?

Explanation:

A
.

The given figure fails to show the symmetry


with respect to .

B
.

C
.

The figure shows symmetry with respect to f,


also the length of arrow is decreasing away
from the charge shows that magnitude E is
decreasing away from line charge.
But problem with this figure is longest lines
must be shown in most crowded region.

D
.

Answer & Explanation

Here we use lines of fixed segments but

different thickness. But this attempt also


makes the region crowded near origin.

C
1, 3 only
.
D
1, 2 and 3
.
Answer & Explanation

Answer: Option B
Explanation:

This figure shows compromise. A


symmetrical distribution of lines (at every
45) shows azimuthal symmetry and
arrowheads show direction.
View Answer Workspace Report
Discuss in Forum

23 The logic function f(A, B, C) =


. 5, 6) can be represented by :

m(0, 2, 4,
Now the Variable Entered Map (VEM) is:

1.

2.

View Answer Workspace Report


Discuss in Forum

3.
A
1 only
.
B
1, 2 only
.

24 In amplitude modulation, carrier signals A


. cos t has its amplitude A modulated in

proportion with message bearing (low


frequency) signal m(t). The magnitude of

m(t) is chosen to be __________ .


A
less than 1
.
B
less than or equal to 1
.

Minimum f(x) = e0 + e0 = 2.

26 The logic realized by the circuit shown in


. figure below is

C
more than 1
.
D
none of these
.
Answer & Explanation

Answer: Option B
Explanation:
For proper recovery of signal |m(t)| 1.

A
F=A-C
.

View Answer Workspace Report


Discuss in Forum

B
F=A+C
.

25 For real values of x, the minimum value of


. the function f(x) exp (x) + exp (- x) is
A
2
.

B
1
.

C
0.5
.

D
0
.

Answer & Explanation

C
F=BC
.
D
F=BC
.
Answer & Explanation

Answer: Option B
Explanation:

Answer: Option A

F = A BC + ABC + A B C + ABC

Explanation:

= B(AC + AC) + B(AC + AC)

f(x) = ex + e-x

B(A C) + B(A C)

f(x) = ex - ex

(A C) (B B)

f(x) = 0 ex - e-x = 0

A C.

f'(x) = ex + e-x +ve for x = 0

View Answer Workspace Report


Discuss in Forum

Thus minimum.

27 An image uses 512 x 512 picture elements.


. Each of the picture elements can take any of

Explanation:

the 8 distinguishable intensity levels. The


maximum entropy in the above image will
be

4 V/100 rpm =

V/rad/sec

A
2097152 bits
.

= 0.0381 V/rad/sec.

B
648 bits
.

View Answer Workspace Report


Discuss in Forum

C
786432 bits
.
D
144 bits
.

29
.

Consider the signal x(n) = 1 + sin

n+3

Answer & Explanation

Answer: Option C
Explanation:
Each picture element can be represented by 3
bits. Thus total entropy = 512 x 512 x 3 =
786432.
View Answer Workspace Report
Discuss in Forum

28 A tachometer has a sensitivity of 4 V/1000


. rpm. Express the gain constant of the

tachometer in the units volts/(rad/sec).


A
0.0489
.
B
0.0381
.
C
2
.
D
none of these
.
Answer & Explanation

Answer: Option B

cos
n + cos
. Then Fourier
series coefficients are __________ .
A
. 1,

B
. 1,

+ j,

C
. 1, 3 -

j,

j,

- j, -

j, 3 +

j, -

j,+

j,

j, -

D
1, 3 + j, 3 - j, - j, + j
.
Answer & Explanation

Answer: Option A
Explanation:

The signal x(n) = 1 + sin

+ cos

n 3 cos

n + cos

is periodic with period N, and we can expand


x[n] directly in terms of complex
exponentials to obtain

A The closed loop system in never


. stable for any value of a a

Collecting terms, we find that

For some positive values of a, the


B
closed loop system is stable, but not
.
for all positive values
C For all positive values of a, the closed
. loop system is stable
The closed loop system is stable for
D
all values of a, both positive and
.
negative

Thus for Fourier series coefficients for this


example are

Answer & Explanation

Answer: Option C

a0 = 1,

Explanation:
a1 =

- j,

a-1 =

+ j,

a2 =

j,

Closed loop system transfer function

Use Routh criteria

a-2 = - j.
View Answer Workspace Report
Discuss in Forum

30 A certain system has transfer function


.

a is a parameter. Consider
the standard negative unity feedback
configuration as shown below.

Thus for all positive value of 'a' this will be


stable.

31 If a signal f(t) has energy E, the energy of the


. signal f(2t) is equal to __________ .
A
E
.

B
E/2
.

C
E/4
.

D
2E
.

Answer & Explanation

Answer: Option B
Which of the following statements is true?

Explanation:

Energy content of a signal x(t), E =


dt

|f(t)|2
gm = 2pfT(C + Cp) = 2 x p x 1.47 x 1010
= (10-14 + 4 x 10-13)

Now, E' =

|f(2t)|2 dz for signal f(2t)

Putting 2t = z, we get

E' =

|f(t)|2 dz =

gm= 38 m.
View Answer Workspace Report
Discuss in Forum

View Answer Workspace Report


Discuss in Forum

32 For a npn BJT transistor f is 1.64 x 108 Hz.


. C = 10-14 F; Cp = 4 x 10-13 F and DC current

gain is 90. Find fT and gm (f = cut off


frequency, C = capacitance, Cp = parasitic
capacitance, gm = transconductance, fT = gain
BW product)
A fT = 1.47 x 1010 Hz ; gm = 38 milli
. mho
B
fT = 1.64 x 108 Hz ; gm = 30 milli mho
.

33 The impulse response h(t) of a linear time. invariant continuous time system is

described by h(t) = exp (at) u(t) + exp (t) u


(- t), where u(t) denotes the unit step
function, and a and are constants. This
system is stable if
A
a is positive and is positive
.
B
a is negative and is negative
.
C
a is positive and is negative
.
D
a is negative and is positive
.
Answer & Explanation

C
fT = 1.47 x 109 Hz ; gm = 38 mho
.

Answer: Option D

D
fT = 1.33 x 1012 Hz; gm = 0.37 m-mho
.

Explanation:

Answer & Explanation

h(t) = e+atu(t) + etu(- t)

Answer: Option A
For h(t) to be stable h(t) dt <

Explanation:

It will happen when a is negative and is


positive.

fT = 90 x 1.64 x 10 = 1.47 x 10 Hz
8

10

View Answer Workspace Report


Discuss in Forum

.
34 The probability density function of a random
. variable x is as shown.

B 4
10 rad/sec
.
C 2
10 rad/sec
.
D
10 rad/sec
.
Answer & Explanation

The value of A is:

Answer: Option D

A
.

Explanation:

B
.
C
.

R1 = (10 + 10) = 20 k

D
.
Answer & Explanation

Answer: Option C
Explanation:

36 An 8 level encoding scheme is used in a


. PCM system of 10 kHz channel BW. The

channel capacity is

=1

A
80 kbps
.

=1
Solve this to get value of A = 1/5.
View Answer Workspace Report
Discuss in Forum

B
60 kbps
.
C
30 kbps
.
D
18 kbps
.
Answer & Explanation

35
.

Answer: Option B
Explanation:
L =

C = 2B log2 M;
8

A 10 rad/sec

M = 8 = 23, log2 M = 3 bits


= 2 x 10 x 103 x 3 = 60 kbps.
View Answer Workspace Report
Discuss in Forum

37 For the power amplifier circuit shown below,


. the maximum power dissipated by both

output transistor is

View Answer Workspace Report


Discuss in Forum

38 For static electric and magnetic fields in an


. homogenous source-free medium, which of

the following represents the correct form of


Maxwell's equations?
A .E = 0
. xB=0
B .E = 0
. .B = 0
C x E = 0
. xB=0
D x E = 0
. .B = 0
Answer & Explanation

Answer: Option D
Explanation:
A
9.66 W
.
B
30.11 W
.

Curl of electric field is zero. Divergence of


magnetic field is zero.
View Answer Workspace Report
Discuss in Forum

C
31.66 W
.
D
33.66 W
.
Answer & Explanation

Answer: Option C
Explanation:
The maximum power dissipated by both
output transistor is maximum

39 A periodic rectangular signal, x(t) has the


. waveform shown in the given figure.

Frequency of the fifth harmonic of its


spectrum is __________

.
D
all pass
.
Answer & Explanation

Answer: Option D
Explanation:
A
200 Hz
.
B
250 Hz
.
C
400 Hz
.
D
1250 Hz
.
Answer & Explanation

Consider an anti-symmetric filter response


with order 5.
h(n) = [h(2), h(1), h(0), - h(1), - h(2)]
We know, H(0) = addition of all component
values.
Hence H(0) = h(0)

Answer: Option D

Similarly H(p) = addition of all component


values with alternate negative sign.

Explanation:

= h(2) - h(1) + h(0) + h(1) - h(2) = h(0)

The periodic time = 4 ms = 4 x 10-3 s

Hence we get all pass filter.

The fundamental frequency


41 A material has conductivity of 105 mho/m
. and permeability of 4p x 10-7 H/m. The skin

depth at 9 GHz is
Frequency of the 5th harmonic
= 250 x 5 = 1250 Hz.
View Answer Workspace Report
Discuss in Forum

40 An antisymmetric filter having odd number


. of coefficient may have a performance as
A
low pass
.
B
high pass
.
C band pass

A
1.678 m
.
B
26 m
.
C
17 m
.
D
32.32 m
.
Answer & Explanation

Answer: Option C
Explanation:

B
.
C

.
D
.
View Answer Workspace Report
Discuss in Forum

Answer & Explanation

Answer: Option B
Explanation:

42 A fair coin is tossed independently four


. times. The probability of the event "the

number of time heads shown up is more than


the number of times tails shown up" is

.
View Answer Workspace Report
Discuss in Forum

A
.
B
.

44 Find RTH across the terminals A and B


.

C
.
D
.
Answer & Explanation

Answer: Option D
Explanation:

A
52.28
.

4 heads and 0 tail 3 head and 1 tail

B
58.28
.

.
View Answer Workspace Report
Discuss in Forum

C
52.82
.
D
58.82
.
Answer & Explanation

Answer: Option D
43 The steady state error of a stable type 0 unity
. feedback system for a unit step function is :
A
0
.

Explanation:
There are not independent source in the
circuit.

The Thevenin and Norton equivalent will


have 0 A and 0 V sources.

A
.

To find RTH, a 1 A source is connected as

B
.

Writing a nodal equations at n,

C
.
D
None of these
.
Answer & Explanation

Answer: Option A
Explanation:
Apply Nodal Analysis

0.01Vx + 0.003Vx + 0.0067Vx = 1


0.017Vx = 1
Vx = 58.82 V
At node V

View Answer Workspace Report


Discuss in Forum

45 Find Y- parameters
.

...(i)
At node V2

...(ii)
At node V3

46 For a second-order system with the closed.

loop transfer function


The
settling time for 2-percent band, in seconds,
is :

Put V3 in equation (i)

A
1.5
.

B
2.0
.

C
3.0
.

D
4.0
.

Answer & Explanation

Answer: Option B
Explanation:

.
View Answer Workspace Report
Discuss in Forum

Put V3 in equation (ii)

47 An open loop transfer function is given by


.

has
A
one zero at
.
B
two zeros at
.
C
three zeros at
.

D
four zeros at
.
Answer & Explanation

Answer: Option C
Explanation:

Answer: Option C
Explanation:
The root locus is given below.

Power = I2R = 25 X 10 = 250 Watts.


View Answer Workspace Report
Discuss in Forum

49 The cut off voltage for JFET is 5 V. The


. pinch off voltage is __________ .
A
(5.0)l/2 V
.
B
2.5 V
.

From root locus, we can see that there are 3


zeros at .
View Answer Workspace Report
Discuss in Forum

C
.

D
5V
.
Answer & Explanation

48 The current i(t) through a 10 resistor in


. series with an inductance is given by i(t) = 3

Answer: Option D

+ 4 sin (100t + 45) + 4 sin (300t + 60)


Amperes. The RMS value of the current and
the power dissipated in the circuit are

Explanation:

A
41 A, 410 W respectively
.

ID = IDSS (1 - VGS/Vp)2

B
35 A, 350 W respectively
.
C
5 A, 250 W respectively
.
D
11 A, 1210 W respectively
.

For JFET

0 = (1 - VGS/Vp)2
VGS = VP
Vp = 5 V.
View Answer Workspace Report
Discuss in Forum

Answer & Explanation


50 Assertion (A): Conductors do not permit

propagation of waves more than a short


distance into the conductor at microwave
frequencies.

SECTION 3

1.The circuit given below is:

Reason (R): The relaxation time constant


for conductors is much small than the period
of centimetric EM wave.
A Both (A) and (R) are true and (R) is
. the correct explanation of (A)
B Both (A) and (R) are true but (R) is
. not the correct explanation of (A)
C
(A) is true but (R) is false
.

A
Band stop filter
.

D
(A) is false but (R) is true
.

B
Band pass filter
.

Answer & Explanation

Answer: Option B

C
Oscillator circuit
.

Explanation:

D
All pass filter
.
Answer & Explanation

Depth of penetration =

, where

= relaxation time.
would be small, if is large, which itself
depends upon relaxation time.

Answer: Option B
Explanation:
The given circuit is Band pass filter

t for conductors is of the order of 10-14 s, for


= 3 x 10-2 cm
period of centimetric EM waves

.
View Answer Workspace Report Discuss
in Forum

2.Rank of the matrix:

Therefore, even though t is much smaller


than period of centimetric waves but it is not
the correct reason for the assertion given.
A
3
.

B
2
.

C1

D4

digital output of the ADC will be

Answer & Explanation

A
1011
.

B
1101
.

Explanation:

C
1100
.

D
1110
.

The given matrix is equivalent to

Answer & Explanation

Answer: Option C

Answer: Option D
Explanation:
,
Thus rank is 1.

Resolution =

View Answer Workspace Report Discuss


in Forum

= 0.5

2n = 14 = (1110)
3.His rather casual remarks on politics

__________ his lack of seriousness about the


subject.
A
masked
.

B
belied
.

C
cherish
.

D
conserve
.

Answer & Explanation

Answer: Option C
Explanation:
The key words in the statement are 'casual
remarks' and 'lack of seriousness'.
The blank should be filled with a word
meaning 'showed' or 'revealed'. Hence,
'betrayed' is the correct answer.
View Answer Workspace Report Discuss
in Forum

4.The resolution of a 4 bit counting ADC is 0.5

volts. For an analog input of 6.6 volts, the

n = 4 bits.
View Answer Workspace Report Discuss
in Forum

5.For the power amplifier circuit shown below,

the maximum power dissipated by both output


transistor is

C
57.61%
.

D
15.24%
.

Answer & Explanation

Answer: Option A
Explanation:
From the transient response characteristics,
Maximum percentage overshoot (MPO) is
given as

A
9.66 W
.

MPO decides the relative stability of the


system.

B
30.11 W
.

View Answer Workspace Report Discuss


in Forum

C
31.66 W
.
D
33.66 W
.

7.Find Vo(t) for the following circuit.

Data: L = 1 H, C = 1 F, R1 = R2 = 1 and
zero initial conditions.

Answer & Explanation

Answer: Option C
Explanation:
The max. power dissipated by both output

transistor is max. P2 =
31.66 W.

6 The maximum percent overshoot with


. maximum value of first overshoot as 3.5 V

and steady state value as 2.575 V is


A
35.92%
.

B
26.43%
.

A
0V
.
B
5V
.
C
3V
.

D
2V
.

Answer: Option C

Answer & Explanation

Explanation:

Answer: Option A
.

Explanation:
E(s) = L[e(t)] = L[5 sin 3t]

View Answer Workspace Report Discuss


in Forum

9.The figure shown below is designed to act as

Let V1(s) = Voltage across L

a constant current source across the load


resistor. The of transistor is 50. What is the
value of load current?

and
V2(s) = Voltage across R2

V(s) = V1(s) - V2(s)

View Answer Workspace Report Discuss


in Forum

8.An XOR gate with 8 variable is as follows:

A BC D E FGH
The number of minterms in the Boolean
expression is

A
1.96 mA
.
B
2 mA
.
C
2.04 mA
.

A
255
.

B
127
.

D
None
.

C
128
.

D
256
.

Answer & Explanation

Answer: Option A

Answer & Explanation

Explanation:

Voltage at base = 12 - 5 = 7 v
Voltage at emitter = 7 + 0.7 = 7.7 v
Current through R1 = 2 mA

Thus current through load =


1.96 mA.

View Answer Workspace Report Discuss


in Forum

The current transfer ratio

, is

A
.
B
.

10 For a given magnetic material operating in


. linear mode B = 0.06 T. Assume

permeability = 51, then magnetization is:


A
44836 A/m
.

C
.
D
.
Answer & Explanation

B
46834 A/m
.

Answer: Option B

C
946 A/m
.

Explanation:

D
936 A/m
.
Answer & Explanation

Answer: Option B
Explanation:

.
View Answer Workspace Report
Discuss in Forum

12 Let f(A, B) = A B; then the simplified


. form of the function f(f(x y, z)w) is

= mH,
m = ( - 1) = 51 - 1 = 50

A
(x y z)w
.

= 50 x H

B
xyzw
.

46834.

11 The circuit is shown in figure.

C
(x y z) + w
.
D
none of these
.

Answer & Explanation

Answer: Option D

C
5
.

Explanation:

D
does not exist
.

f(A, B) = A B
f(x y, z) = (x y z) = M
But f(MW) is undefined.
View Answer Workspace Report
Discuss in Forum

Answer & Explanation

Answer: Option B
Explanation:
x2 = 2x + 1 = 16
x2 + 2x - 15 = 0, (x - 3)(x + 5) = 0, x = 3, - 5

13 A system has 12 poles or 2 zeros. Its high


. frequency asymptote in its magnitude plot

has slope of
A
-200 dB/decade
.
B
-240 dB/decade
.

Thus x = 3.
View Answer Workspace Report
Discuss in Forum

15 For the network shown below, the Thevenin


. voltage and RTH is equal to __________

C
-280 dB/decade
.
D
-320 dB/decade
.
Answer & Explanation

Answer: Option A
Explanation:
High frequency asymptotes = - 20(12) +
2(20) = - 200 dB/DECADE.
View Answer Workspace Report
Discuss in Forum

14 (1 2 1)x = (1 0 0)4 Then x is equal to


.
A
2
.
B3

A
0 V, 58.82
.
B
0 V, 96.774
.
C
0 V, 3 K
.
D
None of these
.
Answer & Explanation

Answer: Option A
Explanation:

Since there is no independent source in the


network; VTH = 0. To find RTH apply 1 A
current source across A and B.

B
.
C
.
D
2s + 3
.
Answer & Explanation

Answer: Option C
Apply KCL at node Vx
Explanation:

as

Solving Vx = 58.82 V

View Answer Workspace Report


Discuss in Forum

16 Impedance function for the network shown


. below is:

17 The function f = A B A D E C
. DEFGFGCEA

DBEEBBBBD
can be written as
A
A BD
.
B
A BE
.
C
AE
.

A
(3s + 1)
.

D
AE
.
Answer & Explanation

Answer: Option D

Sin (x2) will have even powers

Explanation:

Cos (x3) will have even powers

AAB B C D D E E F G

Cos (x2) will have even powers.

21332314122

View Answer Workspace Report


Discuss in Forum

0 AB B 0 D D 0 E 0 0
f =A B B D D E
= A 1 1 E

19 Find the maximum power that can be


. delivered to a load

= A 0 E = A E.
View Answer Workspace Report
Discuss in Forum

18 Which of the following functions would


. have only odd powers of x in its Taylor

series expansion about the point x = 0?


A
sin (x3)
.
B
sin (x2)
.
C
cos (x3)
.
D
cos (x2)
.
Answer & Explanation

A
0.025 W
.
B
2.5 W
.
C
2.5 W
.
D
0.25 W
.
Answer & Explanation

Answer: Option D
Explanation:

Answer: Option A
Explanation:

PL = I2R =

x R (i)

where zS = 100 + j50


zL = 100 - j50
Thus sin(x3) will have odd powers

Putting the values in eq. (i)

PL =

X l00 = 0.25 watts.

View Answer Workspace Report


Discuss in Forum

20 For a integrator circuit, Ri = 100 K, CF =


. 10 F. The input is a step dc voltage as

shown below. Its output will be

D
.

Answer & Explanation

Answer: Option B
Explanation:

A
.

21 The distance between target and radar is


. reduced to half. Power received increases by

B
.

A
1/16
.
B
1/4
.
C
4 times
.
D
16 times
.

C
.

Answer & Explanation

Answer: Option D
Explanation:
.

View Answer Workspace Report


Discuss in Forum

We have zeros at z = 1/3 and z = - 1/2

22 The Causal system given below is


. __________

H(z) = 6 + z-1 - z-2


A
maximum phase
.
B
minimum phase
.

All the zeroes lies inside the unit circle, it


is a min. phase system.

C
mixed phase
.

View Answer Workspace Report


Discuss in Forum

D
none of these
.
Answer & Explanation

Answer: Option B
Explanation:

23
.

Given potential
sin cos , find
flux density D at (2, 2p, 0)

Minimum phase

A
22.12 pC/m2
.

H(z) = 6 + z-1 - z-1

B
2.212 pC/m2
.
C
22.12 nC/m2
.
D
2.212 nC/m2
.

6z2 + z - 1 = 0

Answer & Explanation

Answer: Option A
Explanation:

Now E = -V
E = V

A
(i) only
.

At r = 2, = p/2; = 0 we have

B
(iii) only
.
C
(ii) only
.
D
(i) and (ii)
.
Answer & Explanation

D = 22.12 PC/m .
2

View Answer Workspace Report


Discuss in Forum

24 The following circuit is implementation of


.
1. Sum of full adder
2. Carry of half adder
3. Difference of full subtractor

Answer: Option D
Explanation:

A
1
.
B
.
C
.
D
.
Answer & Explanation

Answer: Option A
Explanation:
For finding Thevenin equivalent
Short voltage source
Open circuit source
Now Thevenin equivalent

.
Now it is evident from the truth tables that
sum and differennce are same.
View Answer Workspace Report
Discuss in Forum

25 The Thevenin impedance Zth between the


. nodes P and Q in the following circuit is

26 If the memory chip size is 256 x 1 bits, then


. the number of chips required to make up 1

kB (1024) bytes of memory is


A
32
.

B
24
.

C
12
.

D
8
.

Answer & Explanation

Answer: Option A
Explanation:

View Answer Workspace Report


Discuss in Forum

i.e. P decreases 4 times.


View Answer Workspace Report
Discuss in Forum

27 A coil has a designed for high Q


. performance at a rated voltage and a

specified frequency. If the frequency of


operation is doubled, and the coil is operated
at the same rated voltage, then the Q factor
and the active power P consumed by the coil
will be affected as follows.
A
P is doubled, Q is halved
.
B
P is halved, Q is doubled
.
C
P remains constant, Q is doubled
.
D
P decreases 4 times, Q is doubled
.
Answer & Explanation

Answer: Option D

28 The number of variables in a function is odd


. and the number of minterms with even

number of 0's are same as the number of


minterms with odd number at 1 's, the
expression formed can be represented in
__________ as well as __________ form.
A
exclusive - OR, NOR
.
B
equivalence, exclusive - OR
.
C
AND, coincidence
.
D
OR, exclusive - OR
.
Answer & Explanation

Answer: Option B
Explanation:
Explanation:
2L = 21L, R remains constant

Q2 =

= 2Q1 i.e. Q is doubled

With odd number of variables (n) terms with


even number of 0's and terms with odd
number of 1 's are the same.
Thus the k-map for the above condition with
minterms represented by terms with even
number of 0's is

for a high a coil L >> R

for n = 3

The expression can be represented by


equivalence (EX NOR) or (EX OR)
expressions.

View Answer Workspace Report


Discuss in Forum

The total number of trees possible


T = NN - 2 = 88 - 2 = 262144.

29 Find the number of loop equation and


. number of possible trees for the given graph.

View Answer Workspace Report


Discuss in Forum

30 The number of open right half plane poles of


.

is
A
0
.

B
1
.

C
2
.

D
3
.

Answer & Explanation

Answer: Option C
A
7, 2097152
.
B
7, 262144
.
C
7.5764801
.

Explanation:
Use Routh criteria.

31 The common emitter model is shown below:


.

D
6, 823543
.

The h-parameter of this model are:

Answer & Explanation

A
.

Answer: Option B
Explanation:
In any graph, Number of loop equations L =
B-N+1
where B = Number of branches, N =
Number of nodes.

B
.
C
.
D
.
Answer & Explanation

Given B = 14, N = 8
L=B-N+1=7

Answer: Option A

Explanation:

Explanation:
During +ve part of Vi
D1 will be forward biased

h parameter
V1 = (3 + 5)I1 + V2

Zener diode is reverse biased


Thus net voltage = 6.8 + 0.7 = 4.5 V

.
View Answer Workspace Report
Discuss in Forum

During -ve part of Vi


D2 will be forward biased
D1 will be reversed biased

32 In the following limiter circuit, an input


. voltage V1 = 10 sin 100pt applied. Assume

that the diode drop is 0.7 V when it is


forward biased. The zener breakdown
voltage is 6.8 V

Thus net voltage = -0.7 V .


View Answer Workspace Report
Discuss in Forum

33 Four signals
.

m1 = cos (0t), m2(t) = cos (0t), m3(t) = 2


cos (0t) and m4(t) = cos (4 0t) are
multiplexed by Time Division Multiplexing
system. The commutator speed in rps is
The maximum and minimum values of the
output voltage respectively are
A
6.1 V, -0.7 V
.
B
0.7 V, -7.5 V
.
C
7.5 V, -0.7 V
.
D
7.5 V, -7.5 V
.

A
2f0
.

B
3f0
.

C
4f0
.

D
8f0
.

Answer & Explanation

Answer: Option D
Explanation:
Apply Nyquist criteria.
Maximum angular speed is 40

Answer & Explanation

Answer: Option C

Thus commutator speed must be 2 x 42 =


80 .

View Answer Workspace Report


Discuss in Forum

34 The feedback control system shown in figure


. is stable for what values of K and T?

B
.
C
.
D
.
Answer & Explanation

Answer: Option A
Explanation:
A
K < 0, T > - 1
.

Use Mesh analysis

B
K > 0, T > 0
.

z parameters are given by,

C
K > 0, T > - 1
.

V1 = Z11I1 + Z12I2 ...(a)


V2 = Z21I1 + Z22I2 ...(b)

D
K < 0, T > 0
.
Answer & Explanation

Answer: Option C
Explanation:
Apply Routh Hurwitz criteria.
View Answer Workspace Report
Discuss in Forum

Apply KVL to mesh 1


V1 = 3I1 - 2I3 ...(i)
Apply KVL to mesh 2
V2 = 2I2 + 2I3 ...(ii)

35 Find z parameters for the following network


.

Apply KVL to mesh 3


5I3 + 2I2 -2I1 =0 ...(iii)
5I3 = 2I1 - 2I2

A
.

Put I3 in eq. (i)

Answer & Explanation

Answer: Option C
Explanation:
Using Mesh analysis
...(iv)
Put in (ii)

Similarly,

...(v)

Compare equation (iv) to (a) and equation


(v) to (b)

Loop 1 : 6 = 9i1 - 3i4 ...(i)


.

Loop 2: 6 + 6 = - 6i2
i2 = - 2A ...(ii)

36 Find the value of ix if each resistance is of 3


. and each battery is of 6 V.

Loop 3: 6 = 9i3 - 3i4 ...(iii)


Loop 4: 6 = 9i4 - 3i3 - 3i1 ...(iv)
Solving the above equations simultaneously
i1 = i3 = 1.1428 A
i4 = 1.432 A
ix = i4 - i3 = 0.2892 A.

A
1.1428 V
.
B
1.432 A
.
C
0.2892 A
.
D
OA
.

View Answer Workspace Report


Discuss in Forum

37 An image uses 512 x 512 picture elements.


. Each of the picture elements can take any of

the 8 distinguishable intensity levels. The


maximum entropy in the above image will
be
A 2097152 bits

.
B
786432 bits
.
C
648 bits
.
D
144 bits
.
Answer & Explanation

Answer: Option B
Explanation:
Picture elements (pixel) = 512 x 512 =
262144
Information per picture element N = log2 M
M = 2N

B
0
.
C
400
.
D
None of these
.
Answer & Explanation

Answer: Option C
Explanation:
Since there is no independent source in the
network
VTH = O V
To find RTh apply : A current source of 1 A
across A and B.

8 = 2N
N=3
Information for 262144 pixels = 262144 x 3
= 786432 bits
Entropy is same as average information per
message.

Apply KCL at node n1

View Answer Workspace Report


Discuss in Forum

38 Find RTH
.

Apply KCL at node Vn2

Vn1 = 2Vx put in (i)

A
200
.

Vn2 = 4vx
put in (ii)

= ei2pft

h(t) . e- j2pft . dt

= ej2pft x H(f) = x(t) x H(f)


100 + Vx = 2Vx

100 = Vx

View Answer Workspace Report


Discuss in Forum

Vn2 = 4Vx = 400 V

40 The dual transform of the given network as


.

View Answer Workspace Report


Discuss in Forum

39
. If

then for this to be true x(t) is


__________ .
A
exp (j2pf/t)
.

A
.

B
exp (-j2pf/t)
.
C
exp (j2pft)
.

B
.

D
exp (-j2pft)
.
Answer & Explanation

Answer: Option C

C
.

Explanation:
Consider x(t) = ej2pft

D
none of the above
.

y(t) = h(t) * x (t)

Answer & Explanation

Answer: Option C
y(t) =

h(t). ej2pf(t - t) . dt

Explanation:

42 Find the power absorbed or delivered by


. each source:

Dual of R G

A
a = -32 b = - 12 c = -80 d = -60
.
41 A /4 long high frequency transmission line
. is terminated into one impedance ZR. If Z0 be

the characteristic impedance of the line, then


input impedance Zin is

B
a = -32 b = - 12 c = 80 d = 60
.
C
a = 32 b = 12 c = -80 d = 60
.
D
a = -32 b = 12 c = 80 d = -60
.

A
ZoZR
.

Answer & Explanation

B 2
Z R/Zo
.

Answer: Option D

C 2
Z 0/ZR
.

Explanation:

D
Infinity
.

Source a : Power : 8 x (4) = - 32 W. Source


A has delivered the power as it is. - ve.

Answer & Explanation

Source b: Source B has absorbed power of +


12 W as power is positive

Answer: Option C
Explanation:

Source c: Source C has absorbed power of 5


x 16 = 80 W
, here

and

Source d: Delivers power of 3 x ix x (5) = 3 x


4 x (- 5) = -60 W
Total power of the network is - 32 + 12 + 80
- 60 = 0 Watts.

[Here ZL = ZR].

View Answer Workspace Report


Discuss in Forum

View Answer Workspace Report


Discuss in Forum

43 The function shown in figure, can represent

a probability density function for A


__________

network is

The value of the load resistance RL is

A
A = 1/4
.
B
A = 1/6
.

A
R/4
.

B
R/2
.

C
R
.

D
2R
.

Answer & Explanation

Answer: Option C

C
A = 1/9
.

Explanation:

D
A = 1/2
.
Answer & Explanation

Answer: Option C
Explanation:
The condition for the probability density
function d(x) is
and is always
non negative or the area under the curve
f(x)is unity.
.
=> A x 6 +

x 2 x 3A = 1

or 9A = 1 or A =

View Answer Workspace Report


Discuss in Forum

View Answer Workspace Report


Discuss in Forum

44 If the transfer function of the following

45 A bipolar differential amplifier uses a


. transistor having 0 = 200, ICQ = 100 mA, |

Adm| = 500 and CMRR = 80 dB. The value of


Rc and RE will be respectively __________ .

A
1.25 k, 1.25 M
.

A
.

B
125 k, 1.25 M
.

B
.

C
1.25 M, 1.25 k
.

C
.

D
3.25 k, 3.25 M
.

D
.

Answer & Explanation

Answer: Option A

Answer & Explanation

Explanation:

Answer: Option D

80 = 20 log CMRR

Explanation:

CMRR = 10000

KCL at node V

But CMRR = 2gmRE

= 1.25
Natural response from s + 2, is Ae-2t

Adm = - gmRC
For Vs = sin 2t, s = 2j

= 1.25 K.

46 System is at rest for t < 0. Vs = sin 2t.


. Determine v(t) for t greater than zero.

Steady state response

Since the system is at rest for t < 0

Now substituting t = 0, we get

View Answer Workspace Report


Discuss in Forum

47 A 300 MHz plane EM wave is propagating


. in free space. The wave is incident normally

on an infinite copper slap. The antenuation


constant for the wave is (copper = 5.8 x 107
mho/ m). The skin depth is
A
0.005 x 10-3 m
.

A
AND
.

B
OR
.

C
NAND
.

D
NOT
.

Answer & Explanation

Answer: Option C
Explanation:

B
0.0038 x 10-3m
.
C
0.0028 x 10-3 m
.
D
0.0024 x 10-3 m
.
Answer & Explanation

Answer: Option B
Explanation:
.
View Answer Workspace Report
Discuss in Forum

48 Following figure represents

The output C is low only when A and B are


both 1.
View Answer Workspace Report
Discuss in Forum

49 For parallel RLC circuit, which one of the


. following statements is NOT correct?
A The bandwidth of the circuit
. deceases if R is increased
B The bandwidth of the circuit remains
. same if L is increased
C At resonance, input impedance is a
. real quantity

At resonance, the magnitude of input


D
impedance attains its minimum
.
value
Answer & Explanation

Answer: Option D
Explanation:

Rationalising,

Bandwidth =
Thus it is maximum at resonance.
Equating imaginary parts of numerator to 0,
View Answer Workspace Report
Discuss in Forum

.
50 The practical resonant circuit is shown in
. above figure. Find the expression for the

resonant frequency o

SECTION 4
A
.
B
.
C
.
D
None of these
.
Answer & Explanation

1.A ramp input applied to a unity feedback

system results in 5% steady state error. The


type number and zero frequency gain of the
system are respectively.
A
1 and 20
.
B
0 and 20
.
C
0 and 1/20
.

Answer: Option B

D
1 and 1/20
.

Explanation:

Answer & Explanation

Answer: Option A
Explanation:

ess = 5% .05
A
no real or complex solution
.

B
exactly two distinct complex solutions
.

View Answer Workspace Report Discuss


in Forum

C
a unique solution
.

2.A fair coil is tossed 10 times. What is the

D an infinite number of complex


. solutions

probability that ONLY the first two tosses will


yield heads?

Answer & Explanation

A
.

Answer: Option A
Explanation:

B
.

Since value lies between - 1 and + 1.


Therefore no real or complex solution exists.

C
.

View Answer Workspace Report Discuss


in Forum

D
.
Answer & Explanation

Answer: Option C

4.

Given

If
, then the value of K is

Explanation:
Use ncr, (p)r (q)n-r [for any two losses yield
head].

A
1
.

B
2
.

C
3
.

D
4
.

Answer & Explanation

But in present case it is required case it is


only for first two tosses. Thus in this case

Answer: Option D
Explanation:

10 times.
View Answer Workspace Report Discuss
in Forum

.
3.The equation sin (z) = 10 has

View Answer Workspace Report Discuss


in Forum

5.Consider the Schmitt trigger circuit shown

, 7.5 to - 7.5 .

6.i1(t) for t > 0 is given by

below. A triangular wave which goes from 12 V to 12 V is applied to the inverting input
of the op-amp. Assume that the output of the
op-amp moves from +15 V to -15 V. The
voltage at the non-inverting switches between

A
2.5 - 2.5e-2t
.
B
2.5 - 1.25e-2t
.
C
1.25 - 1.25e-2t
.
D
None of these
.
A
-12 V and +12 V
.

Answer & Explanation

Answer: Option B

B
-7.5 V and +7.5 V
.

Explanation:

C
-5 V and +5 V
.

The time constant t is given by

D
0 V and 5 V
.
Answer & Explanation

As Req = 4 || 4 = 2

Answer: Option B

i1(t) = iF - (iF - i1)e-t/t

Explanation:
ifinal =

= 2.5 A

V0 is switching between + 15 to - 150


i1(t) = 2.5 - (2.5 - 1.25)e-t/1/2
Thus voltage at non-inverting input switches
between

i1(t) = 2.5 - 1.25e-2t.


View Answer Workspace Report Discuss

in Forum

i1 = 2 A, (4 + R) - 2 X 2 -1 = 0
R=1

7.Find I in 4 resistor.

Using Nodal analysis for loop 2

At node A,
VA - 20 + VA + VA - VB = 0
3VA - VB - 20 = 0
3VA - VB = 20 ...(i)

At node B,
A
1A
.

2VB - 2VA + 2VB + VB = 0

B
0.5 A
.

5VB - 2VA = 0 ...(ii)

C
0.75 A
.

Multiplying (i) by 2 and (ii) by 3

D
0.95 A
.
Answer & Explanation

Answer: Option C

6VA - 2VB = 40
- 6VA - 15VB = 0
13VB = 40
VB 3 V

Explanation:
Apply KVL to first to first loop

6i1 - 2i2 = 10

View Answer Workspace Report Discuss


in Forum

(4 + R)i2 - 2i1 - 2i3 = 0


4i3 - 2i2 = 0
but i3 = 0.5 A
2 - 2i2 = 0
i2 = 1 A, 6i1 - 2 = 10

8.Which of the following Boolean Expression

correctly represents the relation between P, Q,


R and M1?

A
M1 = (P OR Q) XOR R
.
B
M1 = (P AND Q) XOR R
.
C
M1 = (P NOR Q) XOR R
.

A
8 kHz
.

D
M1 = (P XOR Q) XOR R
.

B
50 kHz
.

Answer & Explanation

C
20 kHz
.

Answer: Option D
Explanation:
M1 = Z XOR R
M1 = (X, Y) XOR R
= M1 = [P.Q.(P + Q)]XOR R
= (PQ + PQ) XOR R = (P XOR Q) XOR R.
View Answer Workspace Report Discuss
in Forum

D
10 kHz
.
Answer & Explanation

Answer: Option C
Explanation:
Minimum clock freq. = 2fm
Where fm is highest freq. component = 2 x 10
kHz = 10 kHz.
View Answer Workspace Report Discuss
in Forum

9.A sequential multiplexer is connected as

shown in figure. Each time the multiplexer


receives the clock, it switches to the next
channel from 6 to 1. If the input signals are :
A = 10 cos 2p(4 x 103t)
B = 15 cos 2p(5 x 103t)
C = 20 cos 2p(6 x 103t)
D = 10 cos 2p(10 x 103t)
The minimum clock frequency should be
__________ kHz.

10 The Boolean function realized by the logic

circuit shown is

1.

A
F = m(0, 1, 3, 5, 9, 10, 14)
.
B
F = m(2, 3, 5, 7, 8, 12, 13)
.

A
1
.

C
F = m(1, 2, 4, 5, 11, 14, 15)
.

B
.

D
F = m(2, 3, 5, 7, 8, 9, 12)
.

C
.

Answer & Explanation

D
2
.

Answer: Option D

Answer & Explanation

Explanation:

Answer: Option D
Explanation:
By solving

F = m (2, 3, 5, 7, 8, 9, 15).

1 Find RAB

Answer: Option A
Explanation:
Equivalent is
View Answer Workspace Report Discuss
in Forum

1 For the network shown below, determine fHi


2.and fHO, if, r0 = , Cbe = 36 Pf, Cbc = 4 Pf, Cce

= 1 Pf, Cwo = 8 Pf, Cwi = 6 Pf


RTi = RS || R1 || R2 || Ri = 0.531 K
Ci = Cwi + Cbe + (1 + AV)Cbc
= 6pf + 36pf + (1 - c - 90) 4 pf = 460 pf

RT2 = RC || RL = 4 K || 2.2 K = 1.419 KW


C0 = Cwo + Cce + CMO

where Cwo = wiring capacitance at output side,


Cwi = wiring capacitance at input side
A
738.24 KHz, 8.6 MHz
.
B
781.2 KHz, 10.6 MHz
.
C
838.32 KHz, 9.2 MHz
.
D
938.24 KHz, 8.2 MHz
.
Answer & Explanation

View Answer Workspace Report Discuss


in Forum

13 Complex pole in z-plane is as shown below.

This is the pole diagram for __________ .

Explanation:
In TDM minimum bandwidth is twice the
maximum frequency present. Thus, 6W is
the answer.
View Answer Workspace Report
Discuss in Forum

A
Decaying sinusoidal sequence
.
B
Growing sinusoidal sequence
.
C
Decaying non sinusoidal sequence
.
D
Growing non sinusoidal sequence
.
Answer & Explanation

Answer: Option A

15 Which of the following terms are


. equivalent?
1. A B C
2. A B C
3. A B C

4. A B C
A
1 and 4; 2 and 3
.

Explanation:

B
1 and 3
.

Decaying sinusoidal sequence.

C
2 and 3
.

View Answer Workspace Report


Discuss in Forum

D
1, 2 and 4
.
Answer & Explanation

14 Four messages band limited to W, W, 2W


. and 3W respectively are to be multiplex

Answer: Option A

using Time Division Multiplexing (TDM).


The minimum bandwidth required for
transmission of this TDM signal is

Explanation:

A
W
.

B
3W
.

= A(B C) + A (B C) = A B C

C
6W
.

D
7W
.

Answer & Explanation

Answer: Option C

A B C = A (B C) + A(B C)

A B C = A (B C) + A(B C)
= A(B C) + A(B C)
= A B C.

Using the same logic, we have


16 If 137 + 276 = 435 how much is 731 + 672?
.
A
B
534
1403
.
.
C
1623
.

D
1513
.

Answer & Explanation

Answer: Option C

Using digits sum 1 + 2 = 3

Explanation:

Tens digits sum = 3 + 7 = 10

Given, 137 + 276 = 435

i.e., 10 - 2 and + 1 carry forward Hundreds


digits sum = 1 + 7 + 6 = 14

Adding units digits i.e., 7 + 6 = 13, but given


as 5, which is 13 - 8 and also 1 is carry
forwarded to the tens digit.
i.e., + 1

i.e., 14 - 8 = 6 and one carry forward.


View Answer Workspace Report
Discuss in Forum

17 An astable multivibrator circuit using IC 555


. timer is shown below. Assume that the

circuit is oscillating steadily


Here, 7 + 3 + 1 = 1 i.e., 11 - 8 = 3 and 1 is
carry forwarded to hundred digits

Now, the sum of digits in hundred's place is


1+1+2=4
ie.,

The voltage Vc across capacitor varies


between

Answer & Explanation

Answer: Option D

A
3 V to 5 V
.

Explanation:

B
3 V to 6 V
.

Consider general expression

C
3.6 V to 6 V
.

X(j) =

D
3.6 V to 5 V
.

ea|t| e- jt dt

Answer & Explanation

Answer: Option B

eat e-jt dt +

e-ate-jt dt

Explanation:
Charging and discharging level of capacitor
will be the voltage across it.

This is equal to

Vcc and

X(j) =

e(a - j)t dt +

e -(a + j)t dt

Vcc.

Thus 3V to 6V is the voltage VC across the


capacitor.
View Answer Workspace Report
Discuss in Forum

18 The fourier transform of a double sided


. exponential function

is

Since

View Answer Workspace Report


Discuss in Forum

19 The number of product terms in the


. minimized sum of product expression

A
.

obtained through the following k map is


(where "d" denotes don't care states)

B
.

A
2
.

B
3
.

C
.

C
4
.

D
5
.

D
.

Answer & Explanation

Answer: Option B

Explanation:

A - I =

|A - I| =

=0

Y = A B D + A B D + A C D.

1=

View Answer Workspace Report


Discuss in Forum

f(A) = An = 0I + 1A

Replace A by 1, I by 1
20
.

f() = n = 0 + 1
then |A50| will be

If A =
A
.
B
.
C
.
D
.

(1 - 1002)

Differentiate w.r.t. nn - 1 = 1

1 =
0 = n - x

(1 - 502)

1 =
(1 - 1002)

0 =

[1 - n]

(1 - 50 )

Answer & Explanation

Answer: Option B

An =

Explanation:
An = ?
Every n x n matrix satisfy its characteristic
equation |A - I| = 0 -> eigen vector
An =

[1 - n]

. 2n

of to be 25mV at 300K. Compared to


undoped silicon, the Fermi level of doped
silicon

A
goes down by 0.13 eV
.

A50 =

B
goes up by 0.13 eV
.

|A50| =

(1 - 502).

21 A uniform plane magnetic wave incident


. normally on a plane surface of a dielectric

material is reflected and the percentage of


reflected power is 75% What is VSWR?
A
14
.

B
1.4
.

C
15
.

D
13
.

Answer & Explanation

Answer: Option A
Explanation:

C
goes down by 0.427 eV
.
D
goes up by 0.427 eV
.
Answer & Explanation

Answer: Option C
Explanation:

Use EF - Ev =
Since it is doped with acceptor impurity,
Fermi level will shift down.
View Answer Workspace Report
Discuss in Forum

23 The Fourier series of a real periodic function


. has only
1. Cosine terms if it is even
2. Sine terms if it is even
View Answer Workspace Report
Discuss in Forum

22 Silicon is doped with boron to a


. concentration of 4 x 1017 atoms/cm3.

Assuming the intrinsic carrier concentration


of silicon to be 1.5 x 1010/cm3 and the value

3. Cosine terms if it is odd


4. Sine terms if it is odd
Which of the above statements are
correct?
A
(1) and (4)
.

B
(1) and (2)
.
C
(2) and (4)
.
D
(2) and (3)
.
Answer & Explanation

Answer: Option A

C' = B log2[2SNR] = B log2 SNR + B log22


C' = C1 + B.
View Answer Workspace Report
Discuss in Forum

25 Power gain of antenna equals directive gain


. in VHF and UHF range if and only if

efficiency of the antenna is __________ .


Explanation:
, then it has 0 and
HZ frequency component.
View Answer Workspace Report
Discuss in Forum

24 A communication channel with AWGN


. operating at a signal to noise ratio SNR >> 1

and bandwidth B has capacity C1. If the SNR


is doubled keeping B constant, the resulting
capacity C2 is given by
A
C2 2C1
.
B
C2 C 1 + B
.
C
C2 C1 + 2B
.
D
C2 C1 + 0.3B
.
Answer & Explanation

Answer: Option B
Explanation:
C = B log2[1 + SNR]
C = B log2[SNR]

A
0.25
.

B
0.5
.

C
0.75
.

D
1
.

Answer & Explanation

Answer: Option D
Explanation:
Directive gain and power gain are identical
except that power gain takes into account the
antenna losses. It may be written as
Gp = Gd => efficiency
If = 1 , GP = Gd .

26 For the asymptotic Bode magnitude plot


. shown below, the system transfer function

can be

A
.
B
.
C
.
D
.

Answer & Explanation

Mid-band gain =

Answer: Option A

A
100
.

Explanation:

B
- 100
.

.
View Answer Workspace Report
Discuss in Forum

C
50
.
D
- 50
.
Answer & Explanation

27 A square in which each (i, j) element of


. matrix is equal to the negative of the
th

conjugate complex of (j, i)th element, is


called

Answer: Option B
Explanation:

A
Hermitian matrix
.
B
Skew symmetric matrix
.
C
Conjugate anti-symmetric matrix
.
D
Skew-Hermitian matrix
.
Answer & Explanation

.
View Answer Workspace Report
Discuss in Forum

Answer: Option D
29 The characteristic equation of a feedback
. control system is given by s3 + 5s2 + (K +

Explanation:
When aij = -

This matrix is called skew hermitian matrix.


View Answer Workspace Report
Discuss in Forum

6)s + K = 0
where K > 0 is a scalar variable parameter.
In the root-loci diagram of the system the
asymptotes of the root-locus for large values
of K meet at a point in the s-plane, whose
coordinates are :
A
(- 3, 0)
.

28
.

B
(- 2, 0)
.
C
(- 1, 0)
.

D
(2, 0)
.
Answer & Explanation

Answer: Option C
Explanation:
Apply R-H, and find by auxiliary
equation.
View Answer Workspace Report
Discuss in Forum

__________ .
A
H(ej) = 0 for some
.
B
.
C g[n] = {n[h[n * h[n]} is the impulse
. response of a stable system
D
h[n] has a finite duration
.
Answer & Explanation

Answer: Option D
30 The input 3-dB frequency is :
.
A
100 Hz
.
B
148 Hz
.
C
100 kHz
.
D
None
.
Answer & Explanation

Answer: Option B
Explanation:
Input 3 dB frequency
C = 0.40 F.

31 Consider a stable and causal system with


. impulse response h[n] and rational system

function H(z). Suppose it is known that H(z)


contains a pole at z = 1/2 and a zero some
where on the unit circle. The precise number
and locations of all of the other poles is
unknown.
The following statement which is false is

Explanation:
Statement (d) is false because a finiteduration sequence must have an ROC that
includes the entire z-plane, except possible z
= 0 and/or z =
This is not consistent with having a pole at z
= 1/2.

Statement (b) is true


corresponds
to the value of the z-transform of h[n] at z =
2.
Thus, its convergence is equivalent to the
point z = 2 being in the ROC.
Since the system is stable and causal, all of
the poles of H(z) are inside the unit circle,
and the ROC includes all the points outside
the unit circle, including z = 2.
Statement (c) is true. Since the system is
causal, h[n] = 0 for n < 0.
Consequently, h[n] * h[n] = 0 for n < 0; i.e.,
system with h[n] * h[n] as its impulse
response is causal.

The same is then true for g[n] = n[h[n]].

Explanation:

Furthermore, by the convolution property the


system function corresponding to the
impulse response h[n] * h[n] is H2(z), and by
the differentiation property the system
function corresponding to g[n] is

When improvement in transient state


response is required Lead compensator is
used.

From above equation we can conclude that


the poles of G(z) are at the same locations as
those of H(z), with the possible exception of
the origin.
Therefore, since H(z) has all its poles inside
the unit circle, so must G(z).
It follows that g[n] is the impulse response
of a causal and stable system.

When improvement in steady state response


is required lag compensator is used.
Use of lag-lead compensator increases the
low frequency gain which improves the
steady state while at the same time it
increases bandwidth of the system making
the system response very fast.
View Answer Workspace Report
Discuss in Forum

33 For the collector-feedback amplifier shown


. below the voltage gain Av will be

__________ .

Statement (a) is true because there is a zero


on the unit circle.
View Answer Workspace Report
Discuss in Forum

32 When transient and steady state response


. require improvement; a __________ is used.
A
Lead compensator
.
B
Lag - Lead compensator
.
C
Lag compensator
.

A
-113.07
.

B
-103.07
.

C
-107.07
.

D
-93
.

Answer & Explanation

D
Integrator
.

Answer: Option B

Answer & Explanation

Explanation:

Answer: Option B

B
1
.
C
32, 767
.

= - 103.07
RF -> feedback resistor.
View Answer Workspace Report
Discuss in Forum

D
65, 535
.
Answer & Explanation

Answer: Option B
Explanation:

34 Narrow band FM signal can be represented


. as
A A cos (2pfct) - A sin (2pfct) sin
. (2pfmt)
B A cos (2pfmt) - A sin (2pfct) sin
. (2pfmt)

First bit is sign bit, hence its magnitude its


2s complement
Sign bit is part of magnitude, so it may be
1.

C A cos (2pfct) + A sin (2pfct) sin


. (2pfmt)
D A cos (2pfmt) + A sin (2pfmt) sin
. (2pfct)

36 In the following transistor circuit VBE = 0. 7


. V, rc = 25mV/IE, and and the capacitance

are very large

Answer & Explanation

Answer: Option A
Explanation:
General expression for narrowband FM is
A cos (2pfct) - A sin (2pfct) sin (2pfmt).
View Answer Workspace Report
Discuss in Forum

35 2's complement representation of a 16 bit


. number (one sign bit and 15 magnitude bits)

is FFFF. Its magnitude in decimal


representation is
A
0
.

The value of DC current IE is


A
1 mA
.
B
2 mA
.
C 5 mA

n = 8.

D
10 mA
.

View Answer Workspace Report


Discuss in Forum

Answer & Explanation

Answer: Option A

38 A second-order system has a transfer


.

Explanation:
Design is independent of IB as is very high
IB 0

function is given by
. If
the system, initially at rest is subjected to a
unit step input at t = 0, the second peak in
response will occur at :
A
p sec
.

Thus applying Kirchhoff's law is base


emitter, we get.
View Answer Workspace Report
Discuss in Forum

37 A certain JK FF has tpd = 12 nsec. The largest


. MOD counter that can be constructed from

these FFs and still operate up to 10 MHz is


A
any
.

B
8
.

C
256
.

D
10
.

Answer & Explanation

B
p/3 sec
.
C
p/6 sec
.
D
p/2 sec
.
Answer & Explanation

Answer: Option C
Explanation:

Peak time

Answer: Option B
Explanation:
.
View Answer Workspace Report
Discuss in Forum

f0 = 107 Hz

8.33

39 Value of ZL for maximum power transfer is

.
D
none of these
.
Answer & Explanation

Answer: Option A
A
R
.

Explanation:
Mean value of V2 =

B
R + jL
.
C
R - jL
.

= 965 V

D
None of these
.

rms value =

Answer & Explanation

Answer: Option C
Explanation:

31 V.

41 The magnitude plot of a rational transfer


. function G(s) with real coefficients is shown

below. Which of the following compensators


has such a magnitude plot?

The value of load for maximum power


transfer is given by complex conjugate of
ZAB. ZAB = R + jXL = R + jL.
ZL for maximum power transfer is given
by, ZL = R - jL.
View Answer Workspace Report
Discuss in Forum

40 A periodic voltage has following values for


. equal time intervals changing suddenly from

one value to next : 0, 5, 10, 20, 50, 60, 50,


20, 10, 5, 0, - 5, - 10 etc.
RMS value of the waveform is :
A
31 v
.
B
32 v
.
C insufficient data

A
Lead compensator
.
B
Lag compensator
.
C
PID compensator
.
D
Lead-lag compensator
.
Answer & Explanation

Answer: Option D

y2 = x2 x1 1 1 = 0

Explanation:

y3 = x3 x2 = 0 0 = 0

Since magnitude plot shows both increasing


as well as decreasing plot, it is lead-lag
compensator.

y4 = x4 x3 = 1 0 = 1

View Answer Workspace Report


Discuss in Forum

View Answer Workspace Report


Discuss in Forum

42 The circuit shown in the figure converts


.

if input is gray then y1 y2 y3 y4 will be Binary.

43 Using 4-bit numbers (n = 4) if k = (0011)2


. how is k expressed in 2's complement.
A
(1011)2
.

B
(1101)2
.

C
(1100)2
.

D
(0101)2
.

Answer & Explanation

Answer: Option B
A
BCD to Binary code
.

Explanation:
In 2's complement

B
Binary to excess - 3 code
.

Simply find 2s complement

C
Excess - 3 to Gray
.

(0011)2 1100 + 1 (1101)2 .

D
Gray to Binary
.

View Answer Workspace Report


Discuss in Forum

Answer & Explanation

Answer: Option D
Explanation:
Let input is 1 1 0 1
y1 = x1

44 Which of the following options is the closest


. in meaning to the world below:

Circuitous
A
cyclic
.

B
indirect
.

C
confusing
.

D
crooked
.

Answer & Explanation

Answer: Option B
Explanation:
Circuitous means round about or not direct.
Therefore the closest in meaning will be
indirect.
View Answer Workspace Report
Discuss in Forum

46 A sphere with charge Q was placed in, but


. not touching a larger hollow sphere. If there

is any charge initially on the outer sphere; to


remove it the outer sphere was earthed
momentarily, and then the inner sphere was
removed. The charge remaining on the outer
sphere was then measured. This charge was
found to be __________ to the charge on the
inner sphere.
A
equal and of same again
.

45 System is :
.

B
equal and of opposite sign
.
C
unequal and of same sign
.
D
unequal and of opposite sign
.
Answer & Explanation

Answer: Option B
Explanation:
A
causal
.
B
stable
.
C
non causal
.
D
anticausal
.

This is true for all sizes of the spheres and


for all types of dielectric media between the
spheres.
View Answer Workspace Report
Discuss in Forum

47 At t = ; VR2 and VC will be


.

Answer & Explanation

Answer: Option A
Explanation:
ROC is towards infinity hence the signal is
causal. Now the ROC does not contain the
unit circle hence the system is unstable.

A
4, 4
.
B
2, 4
.
C 0, 2

.
II.

D
2, 0
.
Answer & Explanation

Answer: Option B
Explanation:
When steady state is reached again; then C is
open
Now VR2 = 2V

If segments a to g are considered as


functions of P1 and P2, then which of
the following is correct?
A
g = P1 + P2, d = c + e
.
B
g = P1 + P2, d = c + e
.
C
g = P1 + P2, e = b + c
.

.
View Answer Workspace Report
Discuss in Forum

48 Two products are sold from a vending


. machine, which has two push buttons P1 and

P2. When a button is pressed, the price of the


corresponding product is displayed in a 7segment display. If no buttons are pressed, '0'
is displayed, signifying 'Rs' .0'. If only P1 is
pressed, '2' is displayed, signifying 'Rs. 2'.
If only P2 is pressed, '5' is displayed,
signifying 'Rs. 5'.
If both P1 and P2 are pressed, 'E' is displayed,
signifying 'Error'. The names of the
segments in the 7-segment display, and the
glow of the display for '0', '2', '5' and 'E', are
shown below.

Consider
I.

a segment a to g are considered


as functions of P1 and P2, then
which of the respectively.

push button pressed/not


pressed is equivalent to logic 1/
0 respectively

D
g = P1 + P2, e = b + c
.
Answer & Explanation

Answer: Option B
Explanation:
From the figure

View Answer Workspace Report


Discuss in Forum

49 Laplace transform of {t cos at} is:


.
A
.
B
.
C
.

Explanation:

D
.
Answer & Explanation

Answer: Option A
Explanation:
x(t)

x(s)
.

View Answer Workspace Report


Discuss in Forum

50 The following series RLC circuit with zero


. initial conditions is excited by a unit impulse

function (t).

SECTION 5

1.Determine the potential difference between

the points A and B in the steady state

For t > 0, the output voltage Vc (t) is


A
.
B
.
C
.
D
.
Answer & Explanation

Answer: Option D

A
40 V
.
B
60 V
.
C
25 V
.
D
75 V
.
Answer & Explanation

Answer: Option D
Explanation:

View Answer Workspace Report Discuss


in Forum

2.Find the value of K and velocity constant Kv

so that the maximum overshoot in the unit


step response is 0.2 and the peak time is 1 sec.

2 = 0.259(1 - 2) = 0.259 - 0.2592


1.2592 = 0.259
2 = 0.206
= 0.45

A
12.37, 0.175
.
B
12.86, 2.175
.
C
11.36, 1.175
.

2n = K
n = K and 2n = 1 + KKv

D
1.86, 0.175
.
Answer & Explanation

Answer: Option A

n = K

Explanation:
K = 2n
K = 12.37

1 + 12.37Kv = 2 x 0.45 x 3.52


Kv = 0.175 .

4.In the following network, the switch is closed

at t = 0- and the sampling starts from t = 0.


The sampling frequency is 10Hz.

View Answer Workspace Report Discuss


in Forum

3.The transfer function of a discrete time LTI

system is given by

The expression and the region of convergence


of the z-transform of the sampled signal are
A
.

Consider the following statements:


S1: The system is stable and causal for ROC: |
z| > 1/2
S2: The system is stable but not causal for
ROC: |z| < 1/4
S3: The system is neither stable nor causal for
ROC: 1/4 < |z| < 1/2
Which one of the following statements is
valid?
A
Both S1 and S2 are true
.

B
.
C
.
D
.

|z| < e-5


|z| < e-0.05
|z| < e-0.05
|z| > e-5

Answer & Explanation

Answer: Option C
Explanation:

B
Both S2 and S3 are true
.
C
Both S1 and S3 are true
.

X(Z) =

5.e-0.05n.Z-n

D
S1, S2 and S3 are all true
.
Answer & Explanation

Answer: Option C
Explanation:
S1 true, S2 False (meter stable nor causal) S3
True.

.
View Answer Workspace Report Discuss
in Forum

5.The logic function implemented by the

following circuit at the terminal OUT is


View Answer Workspace Report Discuss
in Forum

7.Consider a control system shown in given

figure. For slight variation in G, the ratio of


open loop sensitivity to closed loop sensitivity
will be given by

A
P NOR Q
.
B
P NAND Q
.
C
P OR Q
.

A
1 : (1 + GH)
.

D
P AND Q
.

B
1 : (1 + GH)- 1
.

Answer & Explanation

C
1 : (1 - GH)
.

Answer: Option D

D
1 : (1 - GH)- 1
.

Explanation:

Answer & Explanation

AND gate.

Answer: Option B
6.In a transmission line terminated with a load

Explanation:

equal to the characteristic impedance, the


reflection coefficient is

Open loop sensitivity = 1

A
0
.

B
+1
.

Closed loop sensitivity =

C
-1
.

D
Infinity
.

Answer & Explanation

Answer: Option A
Explanation:

Hence ratio = 1 : (1 + GH) - 1.


View Answer Workspace Report Discuss
in Forum

8.Consider the signal S(t) shown below :

.
View Answer Workspace Report Discuss
in Forum

View Answer Workspace Report Discuss


in Forum

9.The logic function f(A B, C) = m(0, 2, 4, 5,

6) represented by

The slope of the matched filter output during


the interval
A
8
.

B
12
.

C
16
.

D
18
.

Answer & Explanation

A
i
.
B
i, ii
.

Answer: Option A

C
i, iii
.

Explanation:

D
i, ii, iii
.

Match filter will be

Answer & Explanation

Answer: Option B
Explanation:

The output of this will be convolution of S(t)


with match filter.

View Answer Workspace Report Discuss


in Forum

10 An electrical system and its signal-flow


. graph representation are shown in figure (a)

and (b) respectively

I2 = I1G2

Hence,

...(iii)

From SFG ...(iv)


From (ii), (iii), (iv) we get H.

11 Consider the circuit shown below of 2 : 1


. MUX is given by the function g = ac + bc

The values of G2 and H, respectively are


A
.
B
.
C
.
D
.
Answer & Explanation

Then f is
A
X
.

B
XY + E X Y
.

Answer: Option A

C
XE + XY
.

Explanation:

D
None of these
.

Vi(s) = Z1I1 + Z3(I1 - I2) ...(i)

Answer & Explanation

V0(s) = Z4I2

Answer: Option B

I2(Z3 + Z2 + Z4) = Z3I1 ...(ii)

Explanation:

From signal flow graph

g = ac + bc
min c = 0, g = b and c = 1, g = a

So f = y + ya y x + yg ...(i)
g = x b + x.a = x E + X.o x E
from (i) f = y x + y x E.
View Answer Workspace Report
Discuss in Forum

12 For the given phase lead network, the


. maximum possible phase lead is

Hence sin m =
Hence m = 30.
View Answer Workspace Report
Discuss in Forum

13 Given (224)r =
.

The
value of the radix 'r' is :

A
. sin-1

A
5
.

B
6
.

C
7
.

D
10
.

Answer & Explanation

Answer: Option A

B
30
.

Explanation:

C
45
.

(224)r =

D
60
.
Answer & Explanation

= (13)r x (13)r

Answer: Option B

2r2 + 2r + 4 = (r + 3) (r + 3) = r2 + 6r + 9

Explanation:

r2 - 4r - 5 = 0

The maximum phase lead is given by,

(r - 5) (r + 1) = 0
r = - 1 or r = 5

sin m =
Negative base is not possible. Therefore base
of the equation is 5.
Where

View Answer Workspace Report


Discuss in Forum

14 Which factor determines whether the


. medium is free space, lossless dielectric,

lossy dielectric or good conductor?


A
Attenuation constant
.
B
Constitutive parameters (, , )
.
C
Loss tangent
.

A
2
.
B
1.5
.
C
3
.
D
None of the above
.
Answer & Explanation

D
Reflection coefficient
.

Answer: Option A

Answer & Explanation

Explanation:

Answer: Option C

Reduced Diagram :

Explanation:

Loss tangent
(i) for lossless dielectric, tan 1
(ii) for good conductor, tan >> 1
(iii) for lossy dielectric, tan = 1.
RAB = 3 || 6 = 2
View Answer Workspace Report
Discuss in Forum

15 Find RAB
.

16 In the following circuit, the switch S is


. closed at t = 0 The rate of change of current

(0 +) is given by

B A = 2 sin[sin tana r + sina +


. a ]
C A = 2 sin[sin tana r - sina - a
. ]
D
none of these
.
Answer & Explanation

A
0
.

Answer: Option D

B
.

Explanation:

C
.

Spherical :

Ar = A. a r =

.sin.sin =

Explanation:

A = A. a =

.cossin

At t = 0, inductor acts like a open circuit.

= 2 sin2

Answer & Explanation

Answer: Option B

at t = 0+, inductor path is

We know
open circuit

A = Aa =

cos = 2 sin

V = Is.Rs (at t = 0+)

A=
2sina

Thus

A = 2sin[sin tan a r + sina + a].

View Answer Workspace Report


Discuss in Forum

View Answer Workspace Report


Discuss in Forum

17
.

convert into spherical coordinates.


A A = 2 sin[sin tana r + sina + a
. ]

a r + 2sin2a +

18 Consider the following program segment


. written for 8085 based system
LXI SP, FFFFH
MVI A, FFH
MOV BB, A
K: DCR B
PUSH HL
JNZ K
INR A
LXI H, FFCEH

CPI 20H
LHLD

Explanation:

The contents of accumulator is :


A
CE
.
B
20
.
C
indeterminate
.

Y = (A + D + E + F)(A + C + D + E).

D
none of these
.

View Answer Workspace Report


Discuss in Forum

Answer & Explanation

Answer: Option C
Explanation:
Since contents of HL register pair are
unknown and are continuously ''PUSH''ed on
the stack; retrieval of the same will result in
''Indeterminate'' data.
View Answer Workspace Report
Discuss in Forum

20 A 1000 kHz carrier wave modulated 40% at


. 4000 Hz is applied to a resonant circuit tuned

to a carrier frequency and having Q = 140.


What is the degree of modulation after
transmission through this circuit?
A
0.40
.

B
0.20
.

C
0.27
.

D
0.54
.

Answer & Explanation


19 The reduced form of the function
. Y(A, B, C, D, E, F) = m(0, 1, 2, 3, 4, 6, 7,

8, 9, 12, 14, 15, 16, 17, 18, 19, 20, 22, 23,
24, 25, 28, 30, 31) is given by:
A
A(C D + E F + D E) + D E
.
B
(A + C + D + E)(D + E + F)
.
C
(C + D + E + A)(A + D + E + F)
.
D
A C D + A E F + A(D E)
.
Answer & Explanation

Answer: Option C

Answer: Option C
Explanation:
Resulting depth of modulation

Explanation:

Replacing current and voltage by their


phasors I and 10ejp/b, resp. and
by j =
j1000, we obtain the phasor equation
I + 10-3(j1000I) = 10ejp/6
or I(1 + j1) = 10ejp/6

21 For the circuit, let us assume that R = 1 , L


. = 10-3 H, and V(t) = 10 cos (1000t + 30)V

and the differential equation for I(t) is given


by

Having determined the value of I, we now


find the required solution to be
I(t) = Rc [I ejt] = Rc[7.07e -jp/2 ej1000t]
= 7.07 cos (1000t - 15)A.
View Answer Workspace Report
Discuss in Forum

22 Find equivalent capacitance if each


. capacitance is 1 F.
A
7.07 cos (1000t + 15) A
.
B
70.7 cos (1000t - 30) A
.
C
7.07 cos (1000t - 15) A
.
D
7.07 cos (1000t + 15) A
.
Answer & Explanation

Answer: Option C

A
56 F
.

B
6/5 F
.

C
unit ramp
.

C
6/10 F
.

D
unit parabola
.

D
5/12 F
.

Answer & Explanation

Answer: Option B

Answer & Explanation

Explanation:

Answer: Option B
Explanation:
Use similar method. But for capacitor take
inverse of value of capacitance and then
apply procedure same as in resistance case.
After getting the final answer take inverse of
result to get Ceq

View Answer Workspace Report


Discuss in Forum

In Cuboid case,

24 The RMS value of the signal given below is:


.

For resistance
For inductance
For capacitance

View Answer Workspace Report


Discuss in Forum

23 The Z inverse of the given Z transform is :


.

A
.
B
.
C
.
D
.

A
unit step
.
B
unit impulse
.

Answer & Explanation

Answer: Option B

Explanation:

RMS value

View Answer Workspace Report


Discuss in Forum

25 Use block diagram reduction methods to


. obtain the equivalent T.F from R to C.

A
.
B
.
C
.

26 Find E at P(1, 1, 1) caused by four identical


. 3 nC point charges located at P1(1, 1, 0), P2(-

D
.

1, 1, 0), P3(- 1, - 1, 0) P4(1, - 1, 0)

Answer & Explanation

A
21.58ax + 21.58ay + 12az V/m
.

Answer: Option A
Explanation:

B
6.82ax + 6.82ay + 32.89az V/m
.
C
-6.82ax - 6.82ay - 32.89az V/m
.
D -21.58ax - 21.58ay - 12az V/m
.
Answer & Explanation

Answer: Option B
Explanation:

Which of the following can be the value of


the current source?
E = 6.82ax + 6.82ay + 32.89az V/m
r = ax + ay + az
r1 = ax + ay
r2 = - ax + ay

A
10 A
.
B
13 A
.
C
15 A
.

r3 = - ax - ay

D
18 A
.

and r4 = ax - ay

Answer & Explanation

r - r1 = az;

Answer: Option A

r - r2 = 2ax + az;

Explanation:

r - r3 = 2ax + 2ay + az

Since 12A current is coming from one


source and it is also known that 60V source
is absorbing power i.e. current is flowing
inside 60V source.

r - r4 = 2ay + az

12 = x + 1 => I = 12 - x.
View Answer Workspace Report
Discuss in Forum

= 6.8ax + 6.8ay + 32.9az .


View Answer Workspace Report
Discuss in Forum

27 In the interconnection of ideal sources


. shown in the figure, it is known that the 60V

source is absorbing power

28 For the network shown below, if VD = 12 V

and VGSQ = - 2 V value of RS will be

A
minimum phase
.
B
maximum phase
.
C
mixed phase
.

A
13.35 k
.

D
none of these
.
Answer & Explanation

B
3.35 k
.

Answer: Option A

C
33.31 k
.

Explanation:

D
3.95 k
.
Answer & Explanation

Answer: Option B
Explanation:

It has only zeros at origin. Hence it is


minimum phase system.
VGS = VG - IDRS

View Answer Workspace Report


Discuss in Forum

- 2 = 5.44 - 2.22 X 10-3 X RS


RS = 3.35 K.

30 Find RL for maximum power transfer.


.

View Answer Workspace Report


Discuss in Forum

29 The causal system given below is

__________
A3

.
B
1.125
.
C
4.1785
.

A
.

D
None of these
.
Answer & Explanation

Answer: Option A
Explanation:

B
.

Use Y - transformation

C
.

RAB= (9 || 18) || (9 || 18) = 3 .

31 The probability density function (PDF) of a


. random variable X is as shown below

D
.

The corresponding cumulative distribution


function (CDF) has the form

Answer & Explanation

Answer: Option A
Explanation:

Cumulative distribution function

(Pdf) dx

Then when we integrate the line from (-1, 0)


to (0, 1) we get a parabolic curve.
The maximum value of Pdf can be 1.
View Answer Workspace Report
Discuss in Forum

32 The system with given pole-zero diagram is


. __________

Degree of numerator > degree of


denominator. Also ROC is not defined.
Hence response of the system cannot be
determined.
View Answer Workspace Report
Discuss in Forum

33 For the waveform shown below, the slew


. rate and full power bandwidth with respect to
A
Causal
.

op-amp will be respectively

B
Anti-causal
.
C
Both sided
.
D
Cannot be determined
.
Answer & Explanation

Answer: Option D
Explanation:
Z transform for the pole zero plot will be

A
0.02 V/sec, 1.59 KHz
.
B
0.04 V/sec, 1.59 KHz
.
C
0.02 V/sec, 3.58 KHz
.
D
0.02 V/sec, 1.59 KHz
.

Answer & Explanation

Answer: Option A
Explanation:

D The system has zero initial


. conditions.
Answer & Explanation

Answer: Option C
Explanation:

.
View Answer Workspace Report
Discuss in Forum

35 The impulse response h(t) of a linear


. invariant continuous time system is given by

h(t) = exp (- 2t) u(t), where u(t) denotes the


unit step function. The output of this system
to the sinusoidal input x(t) = 2cos(2t) for all
time t, is
Full power bandwidth

View Answer Workspace Report


Discuss in Forum

34 In the derivation of expression for peak


.

percent overshoot,
,
Which one the following conditions is NOT
required?
A
System is linear and time invariant
.
The system transfer function has a
B
pair of complex conjugate poles and
.
no zeros.
C There is no transportation delay in
. the system.

A
0
.
B -0.25
2
cos (2t - 0.125p)
.
C -0.5
2 cos (2t - 0.125p)
.
D -0.5
2 cos (2t - 0.25p)
.
Answer & Explanation

Answer: Option D
Explanation:
When input is sinusoidal then output is also
sinusoidal with same frequency but
amplitude and phase changes. Thus
amplitude is

Phase is tan-1

= 0.25 p

Thus y(t) = 2-0.5 cos(2t - 0.25p).

A.

0.2

B.

0.3

C.

0.5

D.

1.0

Answer & Explanation

Answer: Option C
Explanation:
36. In the following circuit , the comparator output
in log: "I" if V1 > V2 and is logic "0" otherwise.
The D/A conversion is done as per the relations
VDAC =
2n-2 Volts,
whereb3 (MSB), b2, b1 and b0 (LSB) are the
counter outputs. The counter starts from the
clear state
The magnitude of the error between VDAC and
Vin at steady state in volts is

Magnitude error = 6.5 - 6.2 = 0.3 .

You might also like